You are on page 1of 70

1.

Nano is a unit prefix meaning Select the correct answer using the code given
below:
a) One-millionth
b) One-billionth a) 1 Only
c) One-trillionth b) 2 and 3 only
d) One-quadrillionth c) 3 Only
Ans: b. d) None
Ans: d.
https://en.wikipedia.org/wiki/Nano-
Page 181-182, Indias Struggle for Independence
http://www.thehindu.com/business/iiscs-invention- by Bipan Chandra
powers-up-nanoelectronics-
industry/article8175742.ece Rowlatt Acts February 1919: The acts allowed
certain political cases to be tried without juries and
2. Electronic-beam, Optical and Electro- permitted internment of suspects without trial.
lithography in the Electronics industry are Their object was to replace the repressive
processes that are used for provisions of the wartime Defence of India Act
(1915) by a permanent law. All nonofficial Indian
a) Purification of semiconductor wafers used in members of the council (i.e., those who were not
the manufacture of Nano-Integrated Circuits officials in the colonial government) voted against
b) Deposition/coating a substrate (an underlying the acts. The acts were never actually
substance or layer) with a Nanometre-thick layer of implemented.
any desired metal
c) Etching thin lines and patterns on substrates http://www.thehindu.com/opinion/editorial/gujarat
d) Modification of electrical properties of -must-give-up-terror-bill/article8175731.ece
Graphene
Ans: c. 4. Consider the following statements:

An invention by Bengaluru-based scientists at IISc 1. Under the Indian Telegraph Act, State Home
is all set to make inroads into the billion-dollar Secretaries do not have independent authority to
Nano-electronics industry. This is disruptive authorise interception of telephone calls
because the technology can drastically reduce the 2. The Prevention of Terrorism Act, 2003 is
cost of the existing state-of-the-art e-beam invoked often to file charges against individuals
lithography and optical lithography. This invention suspected of engaging in terrorist activities
is a new way to etch thin lines on a substrate using Which of the statements above is/are correct?
electrodes, termed electro-lithography.
a) 1 Only
This will come in very useful in inscribing, for b) 2 Only
instance, nanometer-scale circuits which make up c) Both 1 and 2
IC chips and minute transistors among others. Once d) Neither 1 nor 2
developed into prototypes, this technique could in Ans: a.
principle reduce the cost of the equipment used
presently from about five crore rupees per piece to Under the Indian Telegraph Act, State Home
merely Rs.15-20 lakhs. Secretaries do authorise telephone taps, but using
power delegated to them by the Centre.
http://www.thehindu.com/business/iiscs-invention-
powers-up-nanoelectronics- Indias repeated experiments with anti-terrorism
industry/article8175742.ece laws have been, by and large, unsuccessful. The
Terrorist and Disruptive Activities (Prevention)
3. With reference to the Rowlatt Bills and Rowlatt Act, 1985, a law considered as draconian as the
Satyagraha of 1919, which of the following Rowlatt Act of the colonial era, and its latter-day
statements is/are incorrect? version, the Prevention of Terrorism Act, 2003, had
been allowed to lapse after it was found that they
1. A Satyagraha Sabha was formed to mobilise were prone to persistent misuse. However, with the
and organise the Satyagraha substantive amendments made to the Unlawful
2. The object of the bills was to replace the Activities (Prevention) Act in 2012, the country
provisions of the wartime Defence of India Act does have an effective law to curb modern-day
1915 by a permanent law terrorism.
3. The acts, as passed by the Imperial Legislative
Council, were never actually implemented
http://www.thehindu.com/opinion/editorial/gujarat
-must-give-up-terror-bill/article8175731.ece

5. With regard to the International Monetary Funds


(IMF) recent Country Quota reforms, which of the
following statements is/are correct?

The BRIC nations will be among the ten largest


members alongside large advanced economies in
terms of voting share in the IMF
The combined capital that IMFs 188 member-
countries contribute to it is set to double
Select the correct answer using the code given
below:

a) 1 Only
b) 2 only
c) Both 1 and 2
d) Neither 1 nor 2
Ans: c.

The greatest gains from the reforms accrue to the


IMF itself, as the combined capital that its 188
member-countries contribute will increase to
approximately $659 billion from nearly $329
billion. Other winners are India and China, who
have respectively increased their voting shares by
0.292 and 2.265 percentage points. The emerging
economies wrested a 2.6 percentage points increase.
The developed nations have had a haircut in their
voting share, somewhere between 0.2 and 0.5
percentage points. Consequently, India, China,
Brazil, and Russia will be among the 10 largest
members alongside large advanced economies.

http://www.thehindu.com/opinion/editorial/on-
imfs-country-quota-reforms/article8175732.ece
1. Which of the following are techniques used to Reason (R): Article 361 of the Constitution gives
measure distance of celestial objects from earth? the Governor protection from legal action for acts
done in performance of their powers and duties
1. Parallax method
2. Cepheid Variable Technique Select the correct answer:
3. Spitzer Fixed Technique
4. Red Shift a) A and R both are true, and R is the correct
Select the correct answer using the code given explanation for A
below: b) A and R both are true, and R is the NOT the
correct explanation for A
a) 1 and 4 Only c) A is incorrect, R is correct
b) 2 and 3 Only d) A and R both are incorrect
c) 1, 2 and 4 only Ans: c.
d) 2, 3 and 4 only
The turn of events was triggered by Attorney-
Ans: c. General Mukul Rohatgi, for the Centre, drawing the
Benchs attention to Article 361 (1) of the
Nearby stars are measured with parallax method. Constitution which gives the President and the
The Cepheid variable technique is one such method Governor protection from legal action. Under the
which is useful for stars away millions of light- Article, both the President and the Governor of a
years. In the case of distant galaxies, in the order of State shall not be answerable to any court for acts
billions of light years away, distance is measured done in performance of their powers and duties.
with the help of supernova. Red shift is another
technique used for very far objects. To prove his point, the AG referred to the
Constitution Bench judgment of 2006 in the Bihar
http://www.thehindu.com/sci-tech/outer-space- Assembly dissolution case concerning Governor
measurement/article8172262.ece Buta Singh.

2. Consider the following statements: Senior advocate T.R. Andhyarujina, for the
Governor, said: Neither the President nor the
1. The ratio of algae-to-corals (area of occupation) Governor should be issued notice. However, such
is an indicator of coral reef health immunity to the Governor does not take away the
2. A more productive reef releases more oxygen power of the Supreme Court to examine the validity
Which of the statements above is/are correct? of his action on the ground of mala fide. His
recommendations and their relevance to the
a) 1 Only imposition of Presidents rule would be scrutinised
b) 2 Only by the court.
c) Both 1 and 2
d) Neither 1 nor 2 http://www.thehindu.com/todays-paper/supreme-
Ans: c. court-recalls-notice-to-
governor/article8180663.ece
As coral is degraded, it is replaced by algae and
hence the ratio of algae to coral is an indicator of 4. What are some of the areas of concern for India
reef condition. which will arise due to implementation of the
recently agreed Trans Pacific Partnership?
Measuring the oxygen concentration is very
important as productivity is indicated by the change 1. Labour laws
in oxygen over time for a given area of the reef. A 2. Investor-State dispute settlement mechanism
more productive reef releases more oxygen. 3. Intellectual Property Rights
Select the correct answer using the code given
http://www.thehindu.com/sci-tech/nasas-airborne- below:
survey-of-coral-reefs/article8172257.ece
a) 1 and 2 Only
3. In the context of events unfolding in the Arunachal b) 2 Only
case, consider the following statements: c) 2 and 3 Only
d) 1,2 and 3
Assertion (A): A State Governors
recommendations and their relevance to the Ans: d.
imposition of Presidents rule cannot be scrutinised
by a court of law
These are issues on which India has been in the http://www.thehindu.com/sci-tech/nasas-airborne-
news often. survey-of-coral-reefs/article8172257.ece

http://www.thehindu.com/news/national/transpacif
ic-pact-may-impact-exports-says-commerce-
minister-nirmala-sitharaman/article8180054.ece

5. The theme of World Wetlands Day 2016 is

(a) Wetlands and Agriculture


(b) Wetlands, Biodiversity and Climate Change
(c) Wetlands for our Future: Sustainable
Livelihoods
(d) Wetlands and Water Management

Ans: c.

Wetlands for our Future: Sustainable Livelihoods is


the theme for World Wetlands Day in 2016. This
theme is selected to demonstrate the vital role of
wetlands for the future of humanity and specifically
their relevance towards achieving the new
Sustainable Development Goals.

World Wetlands Day is celebrated every year on 2


February. This day marks the date of the adoption
of the Convention on Wetlands on 2 February 1971,
in the Iranian city of Ramsar on the shores of the
Caspian Sea.

http://www.thehindu.com/sci-tech/energy-and-
environment/states-to-get-greater-role-in-wetland-
management/article8180052.ece

6. National Aeronautics And Space Administrations


CORAL stands for

(a) Coral Reef Alliance


(b) Council of Research and Astrophysics
Laboratory
(c) Coral Reef Airborne Laboratory
(d) Collaborative Research on Absolute Lunation

Ans: c.

As a first step to estimate the extent of damage to


coral reefs, NASA has embarked on an air-borne
three-year field experiment called The COral Reef
Airborne Laboratory (CORAL), which aims to
survey the conditions of the major reefs of the world
through remote-sensing.

Dr. Eric Hochbergs team will survey the


condition of entire reef systems in Hawaii, Palau,
the Mariana Islands and Australia. The regions were
chosen because they represent a wide range of reef
types (fringing, barrier, atoll, oceanic and
continental) and a wide range of environmental
conditions (from pristine to heavily impacted).
1. Which of the following systems of healthcare is - The responsibility of determining whether an
NOT indigenous to India? event is within this category lies with the
WHO Director-General and requires the
a) Yoga convening of a committee of experts the
b) Siddha IHR Emergency Committee.
c) Yunani
d) Ayurveda http://www.thehindu.com/opinion/editorial/on
-the-zika-virus-threat-and-measures-to-
Ans: c. eliminate-it/article8184727.ece

Introduced by Arabs and Persians to India in 3. The Supreme Court case which overturned the
the 12th century and with a rich literature, Delhi High Courts 2009 judgement reading
Unani medicine remains popular in parts of down Section 377 to restrict its criminal import
South and Central Asia. to non-consensual sexual acts involving adults
and all sexual acts inflicted on minors is better
http://pib.nic.in/newsite/PrintRelease.aspx?reli known as the
d=136000
a) Prakash Singh case
2. The Zika virus outbreak has been declared as a b) Lily Thomas case
Public Health Emergency of International c) Naz Foundation case
Concern (PHEIC) by the World Health d) Suresh Kumar Kaushal case
Organisation (WHO). On the basis of which of
the following conditions does the WHO declare Ans: d.
disease outbreaks to be PHEIC?
Two years ago, the Supreme Court declined to
1. When prevalence of the disease in question is review its retrograde decision of 2013
reported by twenty or more countries upholding the validity of Section 377. There is
2. When such disease outbreaks potentially new hope that the Delhi High Court judgment
require a coordinated international response of 2009, reading down Section 377 to restrict
Select the correct answer using the code given its criminal import to non-consensual sexual
below: acts involving adults and all sexual acts
inflicted on minors, may be restored.
a) 1 Only
b) 2 Only http://www.thehindu.com/opinion/editorial/edi
c) Both 1 and 2 torial-on-section-377-curative-
d) Neither 1 nor 2 petition/article8184726.ece

Ans: b. 4. Which of the following statements associated


with the Zika Virus is NOT correct?
The term Public Health Emergency of
International Concern is defined in the a) In majority of the cases, the infection
International Health Regulations (IHR) as remains asymptomatic
an extraordinary event which is determined, b) It is the main cause for microcephaly
as provided in these Regulations: c) The virus could possibly be spreading
through bodily fluids
- to constitute a public health risk to other d) It was discovered in the Zika forest in
States through the international spread of Uganda
disease; and
- to potentially require a coordinated Ans: b.
international response. This definition
implies a situation that: is serious, unusual or From the given statements, b is wrong.
unexpected; carries implications for public
health beyond the affected States national The World Health Organization has declared
border; and may require immediate that the outbreak of Zika and congenital
international action. malformations and neurological disorders in
newborns believed to be connected to the virus
is a global public health emergency. 6. Which of the following is NOT one among the
three major classical texts of Ayurveda?
How might Zika cause brain damage in infants?
a) Charaka Samhita
The possibility that the Zika virus causes b) Sushruta Samhita
microcephaly unusually small heads and c) Ashtanga Hridayam
damaged brains emerged only in October, d) Gheranda Samhita
when doctors in northern Brazil noticed a surge
in babies with the condition. Ans: d.

It may be that other factors, such as The multi-cultural origins of Ayurvedic


simultaneous infection with other viruses, are knowledge are revealed in the classical texts
contributing to the rise; investigators may even themselves. Both Charaka Samhita and
find that Zika virus is not the main cause, Sushruta Samhita urge physicians to seek the
although right now circumstantial evidence help of cowherds, hunters and forest-dwellers
suggests that it is. for procuring medicinal plants. In the Charaka
Samhita, we notice the participation and
http://www.thehindu.com/opinion/editorial/on contribution of a Central Asian physician in
-the-zika-virus-threat-and-measures-to- one of the assemblies of scholars gathered to
eliminate- formulate the principles of Ayurveda.
it/article8184727.ece?ref=sliderNews
The three major classical texts give
5. Which of the following statements is/are NOT importance to Buddhist moral values.
feature(s) of the American Presidential system Vagbhata, who is said to be the author of one of
of government? the classical texts Ashtangahridayam, was a
Buddhist.
1. The President is elected directly by the
citizens http://pib.nic.in/newsite/PrintRelease.aspx?reli
2. The President governs with the help of a d=136000
cabinet which consists of non-elected
members
3. The President and his secretaries are not
responsible to the Congress (the Senate and https://www.ayurveda.com/online_resource/an
the House of Representatives) for their acts cient_writings.html
Select the correct answer using the code given
below:

a) 1 Only
b) 2 and 3 only
c) 3 only
d) 1, 2 and 3

Ans: a.

General question associated with the start of


US Presidential Primaries.

The President is elected by an Electoral


College. The Electoral College consists of
members who are directly elected by the
citizens. Statements 2 and 3 are correct.

Parliamentary System, Indian Polity 4th


Edition by M Laxmikanth
1. Consider the following statements: http://www.thehindu.com/news/national/genet
ic-engineering-appraisal-committee-gm-
1. Inactivated vaccines consist of disease mustard-study-to-be-out-
causing microbes killed with heat, radiation soon/article8189436.ece
or chemicals
2. Most inactivated vaccines stimulate a 3. Consider the following statements about the
weaker immune system response than do Committee of Privileges, a type of
live vaccines Parliamentary Committee:
Which of the statements above is/are incorrect?
1. Its functions are semi-judicial in nature
a) Only 1 2. It can summon non-members of the
b) Only 2 Parliament for hearings in case of breach of
c) Both 1 and 2 privileges of the House and its members
(d) Neither 1 nor 2 3. It consists of 25 members (10 from the
Rajya Sabha and 15 from the Lok Sabha)
Solution: d. Which of the statements above is/are correct?

Scientists produce inactivated vaccines by a) 1 Only


killing the disease-causing microbe with (b) 1 and 2 only
chemicals, heat, or radiation. Such vaccines are c) 2 and 3 Only
more stable and safer than live vaccines: The (d) 1, 2 and 3
dead microbes cant mutate back to their
disease-causing state. Inactivated vaccines Solution: b.
usually dont require refrigeration, and they can
be easily stored and transported in a freeze- The Committees are separate there is one for
dried form, which makes them accessible to the Rajya Sabha and another for the Lok Sabha.
people in developing countries.
Page 22.33, 22.35 Indian Polity 4th edition by
Most inactivated vaccines, however, stimulate M Laxmikanth
a weaker immune system response than do live
vaccines. So it would likely take several http://www.thehindu.com/news/national/rajya-
additional doses, or booster shots, to maintain a sabha-panel-summons-prashant-
persons immunity. This could be a drawback bhushan/article8189435.ece
in areas where people dont have regular access
to health care and cant get booster shots on 4. Majority portion of the fiscal deficit of the
time. Centre is devoted to borrowing for
consumption, in the form of revenue deficit
http://www.thehindu.com/business/bharat- (RD). In this context, which of the following
biotech-to-start-preclinical-trials-soon-on-two- statements is/are incorrect?
vaccine-candidates-for-zika-
virus/article8188474.ece 1. A RD implies that the government is using
up the savings of the other sectors of the
2. The Genetic Engineering Appraisal Committee economy to finance a part of its
is constituted under the consumption expenditure
2. Grants given to state governments, even if
(a) Food Safety and Standards Act, 2006 it may be meant for creation of assets, is a
(b) Geographical Indications of Goods form of revenue expenditure (RE)
(Registration and Protection) Act, 1999 3. Subsidies, salaries and pensions fall under
(c) Environment (Protection) Act, 1986 the head of non-plan RE
(d) Wildlife (Protection) Act, 1972 Select the correct answer using the code given
below:
Solution: c.
a) 1 and 3 Only
CS(P) 2015 b) 2 Only
(c) 1, 2 and 3
d) None Water Saved, Water Produced) is being
prepared for every Jal gram which would
Solution: d. provide the yearly status/information on
availability of water for the village from all
RE relates to those expenses incurred for the sources.
normal functioning of the government
departments and various services, interest http://pib.nic.in/newsite/PrintRelease.aspx?reli
payments on debt incurred by the government, d=136060
and grants given to state governments and other
parties (even though some of the grants may be http://timesofindia.indiatimes.com/city/jaipur/
meant for creation of assets). Uma-Bharti-launches-Jal-Kranti-
Abhiyan/articleshow/47560221.cms
When the government incurs a revenue deficit,
it implies that the government is dissaving and 6. Seema Darshan was a 5-day programme
is using up the savings of the other sectors of organised by the
the economy to finance a part of its
consumption expenditure. This situation means (a) Ministry of Defence
that the government will have to borrow not (b) Ministry of Home Affairs
only to finance its investment but also its (c) Ministry of Human Resource
consumption requirements. Development
(d) a, b and c
Page 62,65 Macroeconomics Class XII
NCERT Solution: d.

http://www.thehindu.com/todays-paper/tp- The Ministry of Human Resource


business/govt-must-absolutely-not-relax-on- Development in collaboration with the
deficit/article8190210.ece Ministry of Defence and the Ministry of Home
affairs organized Seema Darshan programme
5. Jal Kranti Abhiyan is administered by the for students for a period of five days starting
from 22nd January 2016.
(a) Ministry of Water Resources
(b) Ministry of New and Renewable Energy Seema Darshan is a unique initiative
(c) Ministry of Environment and Forests being taken up by the Ministry of Human
(d) Ministry of Agriculture Resource Development, in which school
children are visiting the border areas of our
Solution: a. country and present before our soldiers and
troops an array of performances showcasing the
The Ministry of Water Resources, River tradition and culture of our country through
Development and Ganga Rejuvenation initiated traditional songs and dances. Child artists
Jal Kranti Abhiyan during 2015-16 for creating would demonstrate to the jawans how they are
awareness on aspects of water security and preserving and taking forward the cultural
water conservation. Totally 1348 villages have legacy of that country for which the armed
to be identified in 674 districts, out of which forces are constantly on the vigil and also
1001 have been selected as Jal Grams in the protecting and guarding its borders
country.
http://pib.nic.in/newsite/PrintRelease.aspx?reli
From each Jal Grams, one elected d=136072
representative of Panchayat and one
representative of the water users association are
being identified as Jal Mitra/ Neer Nari and
training is being imparted to them to create
mass awareness about issue pertaining to water
as well as providing necessary guidance in
tackling water supply related routine issues. A
card known as Sujalam Card (with the logo
1. The Government may be looking at big ticket (b) Kolkatta
disinvestment this budget, which includes c) Guwahati and Shillong
Strategic Sales. Which of the following d) Bangalore and Mysore
statements about Strategic Sale is/are
incorrect? Solution: c.

1. 51% or more than 51% of the total equity From a historical perspective, the South Asian
goes to the Strategic Partner (SP) Games were established as a movement to
2. The Governments shareholding goes foster peace and friendship through regular
below 51% sporting contact between athletes from
3. The Government has to ensure that the Afghanistan, Bangladesh, Bhutan, Maldives,
Agreements signed with the SP adequately Nepal, Pakistan, Sri Lanka and current host
safeguard the employees and nations India. The first edition was held in Kathmandu
interest in 1984. That the event is being hosted for the
Select the correct answer using the code given first time in the North East of the country does
below: add significance to the Games, the curtains to
which will be brought down on February 16.
a) 1 Only
b) 1 and 2 Only This is the third time India will host the
c) 3 Only Games, after Kolkata in 1987 and Chennai in
d) None 1995, and it will have hopes of continuing with
its overall dominance across venues for a
Solution: a. twelfth straight time.

Strategic Sale: 3. Consider the following statements:

The transfer of shares by Government may not 1. Between 2001 and 2011, employment
necessarily be such that more than 51% of the growth was lower than growth in the labour
total equity goes to the Strategic Partner for the force
transfer of management to take place. In the 2. Employment elasticity is a measure of how
case of PSUs, in order that the company no employment growth varies with the growth
longer has the character of a Government in labour force
company, the transfer of shares involves Which of the statements above is/are correct?
bringing down Governments shareholding
below 51%. The Strategic Partner, after the a) 1 Only
transaction, may hold less percentage of shares b) 2 only
than the Government but the control of c) Both 1 and 2
management would be with him. For instance, d) Neither 1 nor 2
if in a PSU the shareholding of Government is
51% and the balance is dispersed in public Solution: a.
holdings, then Government may go in for a
25% strategic sale and pass on management The huge challenge of employment generation
control, though the Government would post- needs to be addressed. As the Economic Survey
transfer have a larger share holding (26%) than 2014-15 shows, regardless of the data source
the Strategic Partner (25%). used, employment growth (1.40 per cent) has
lagged behind growth in the labour force (2.23
http://www.thehindu.com/news/national/union per cent) between 2001 and 2011. Clearly,
-budget-2016-bigticket-divestment- employment elasticity of growth, showing the
likely/article8194898.ece effectiveness of the economic system in
generating employment, seems to have
2. The South Asian Games is being held in India declined over time.
for the third time since its inception. It is being
hosted by Employment elasticity is a measure of how
employment varies with economic output.
(a) Delhi
Example (From 2012): The Planning (b) 2 and 4 Only
Commission says that employment elasticity (c) 2,3 and 4 Only
has come down from 0.44 in the first half of (d) 1,2,3 and 4
the decade 19992000 to 200405, to as low as
0.01 during the second half of the decade 2004 Solution: d.
05 to 200910. An employment elasticity of
0.01 implies that with every 1 percentage point Shri Singh said on the basis of the report of the
growth in GDP, employment increases by just Bhuria Committee submitted in 1995, PESA
one basis point. (One basis point is one- was enacted in 1996 to extend Part IX of the
hundredth of a percentage point.) Its as good Constitution with certain modifications and
as saying that the extraordinary growth during exceptions to Fifth Schedule areas in 108
those years didnt lead to any employment districts in 10 States.
growth at all.
Panchayati Raj, Indian Polity by M
http://www.thehindu.com/opinion/lead/going- Laxmikanth
against-the-grain/article8194370.ece

4. With reference to the 73rd amendment to the


Indian Constitution, which of the following
statements is/are incorrect?

1. To reserve seats (both members and


chairpersons) for SCs, STs, OBCs and
women in Panchayats at all three levels is a
compulsory provision
2. The PESA Act, 1996 is applicable only to
Fifth Schedule Areas
Select the correct answer using the code given
below:

a) 1 Only
b) 2 Only
(c) Both 1 and 2
d) Neither 1 nor 2

Solution: a.

Providing reservation of seats for OBCs in


Panchayats at any level is a voluntary
provision.

http://pib.nic.in/newsite/PrintRelease.aspx?reli
d=136091

5. Consider the following list:

1. Bhuria Committee
2. G V K Rao Committee
3. Ashok Mehta Committee
4. Balwant Rai Mehta Committee
Which of the above are associated with the
scheme of democratic decentralisation or as it
has come to be known, Panchayati Raj?

(a) 3 and 4 Only


1. Mera Gaon Mera Gaurav is a programme iron and zinc rich wheat, pearl millet and lentil
under which varieties through molecular breeding, and
genetically modified Golden rice enriched
a) Every Member of each States/Union with pro-vitamin A.
Territorys Legislative Assembly will adopt
one village for developing the socio-economic Hidden hunger is a lack of vitamins and
and physical infrastructure by 2019 minerals. Hidden hunger occurs when the
b) Scientists of the Indian Council of quality of food people eat does not meet their
Agriculture Research will adopt villages and nutrient requirements, so the food is deficient
assist farmers by undertaking agriculture in micronutrients such as the vitamins and
extension activities minerals that they need for their growth and
c) Each village will pick a child aged between development. 2 billion people suffer from
10 and 15 to suggest one developmental work vitamin and mineral deficiencies. Women and
to be carried out for that month in the village children in families with low-income often
d) Every Gram Sabha will hold a bi-monthly dont get enough vitamin a, iodine and iron, and
meeting to discuss cleanliness and hygiene sometimes other essential nutrients. This limits
issues, and evaluate progress made since the their growth, development, health and working
last meeting capacity.

Solution: b. http://pib.nic.in/newsite/PrintRelease.aspx?reli
d=136123
The Mera Gaon Mera Gaurav programme of
IARI, under which each scientist will adopt a 3. The city of Pragjyotishpur in Ancient India
village, should aim at changing agriculture refers to which city today?
from subsistence farming to commercial farm
industry. a) Amaravati
b) Kusinagar
http://pib.nic.in/newsite/PrintRelease.aspx?reli c) Guwahati
d=136123 d) Unakoti

2. Consider the following statements: Solution: c.

1. Although India has succeeded in Guwahati has come a long way since the days
developing higher-yielding varieties of of Pragjyotishpur in Ancient India. It is now a
crops, seeds of bio-fortified crops to this modern and vibrant city and a hub of all
day are imported economic activities in the North-Eastern
2. Hidden hunger occurs when the quality of Region of India.
food people eat does not meet their nutrient
requirements http://pib.nic.in/newsite/PrintRelease.aspx?reli
Which of the statements above is/are incorrect? d=136136

a) 1 Only 4. Which among the following states does NOT


b) 2 Only consist of cities that are on the heritage city list
c) Both 1 and 2 under the Heritage City Development and
d) Neither 1 nor 2 Augmentation Yojana (HRIDAY)?

Solution: a. a) Telangana
b) Andhra Pradesh
Forty-five percent of children below the age of c) Kerala
three years are under-nourished in India. A d) Gujarat
large number also suffer from Vitamin-A
deficiency. To address malnutrition in children, Solution: c.
food grains must be bio-fortified with quality
protein and micronutrients. I am happy that As a pilot, this flagship scheme of the
IARI has developed quality protein maize, and government focuses on revitalising twelve
heritage cities (Click to see Map), namely Amareswaram. The recorded history of
Amaravati (Andhra Pradesh), Gaya (Bihar), Amaravati and nearby Dharanikota is from 2nd
Dwaraka (Gujarat), Badami (Karnataka), Puri century BCE. It was the capital of Andhra
(Odisha), Amritsar (Punjab), Ajmer Satavahanas who ruled from 2nd century BCE
(Rajasthan), Kanchipuram and Velankanni to 3rd century CE.
(Tamil Nadu), Warrangal (Telangana) and
Mathura and Varanasi (Uttar Pradesh). According to Vajrayana traditional sources,
the Buddha preached at
http://pib.nic.in/newsite/PrintRelease.aspx?reli Dharanikota/Dhanyakatakam and conducted
d=136132 Kalachakra ceremony, which would take the
antiquity of Amaravati back to 500 BCE.
5. It was once known as Omtikonda or Ekashila
Nagaram. The thousand pillar temple and the Go through the information about cities on the
Kush Mahal can be found here. It was once HRIDAY list as and when time permits.
under the rule of the Delhi Sultanate. This city
today is known as http://pib.nic.in/newsite/PrintRelease.aspx?reli
d=136132
a) Golkonda
b) Warangal
c) Bijapur
d) Solapur

Solution: b.

Warangal is a city in south-eastern India and


the administrative headquarters of Warangal
district in the state of Telangana. The 11th-14th
century inscriptions and literature refers
Warangal as Omtikonda or Orugallu or
Oruganti or Ekashila Nagaram, on account of
its proximity to a monolithic hillock.

Go through the information about cities on the


HRIDAY list as and when time permits.

http://pib.nic.in/newsite/PrintRelease.aspx?reli
d=136132

6. This was the capital of the Satavahanas and was


once known as Punyakshetra or Amareswaram.
According to Vajrayana traditional sources, the
Buddha preached at a place close-by. It is
located on the banks of River Krishna. This city
today is known as

a) Amaravati
b) Chitradurga
c) Kanaganahalli
d) Nagarjunakonda

Solution: a.

Amaravati is a city located in Guntur district


in Indias eastern state of Andhra Pradesh. The
city is also known as Punyakshetra or
1. Which of the following categories are
applicable for funds under the Priority Sector Why is India opposed to being included in
Lending norms? SRAP?

1. Education Inclusion of India in SRAP would make the US


2. Export Credit third party in India-Pak disputes.
3. Renewable Energy It would also make India indirectly support US
4. Road Sector policy on Afghanistan.
Select the correct answer using the code given http://www.insightsonindia.com/2016/02/01/in
below: sights-daily-current-events-01-february-2016/

a) 1, 2 and 3 only 3. The Pre-Conception and Pre-Natal Diagnostic


b) 2 and 4 Only Techniques Act, 1994 (PCPNDT Act)
c) 1 and 3 only
d) 1, 2, 3 and 4 a) Prohibits ultrasonography tests on a
pregnant lady
Solution: a. b) Permits ultrasonography tests on a pregnant
lady but with certain conditions
The Road Transport and Highways Ministry c) Finds no mention in it about
has decided to approach the Reserve Bank of ultrasonography
India (RBI) Governor Raghuram Rajan with a d) None of the above
proposal to include road projects under the Incorrect
priority sector list for lending purposes. Solution: b.

Categories under Priority Sector Currently Indias strategy to combat declining


sex ratios hinges on the PCPNDT Act (the Pre-
http://www.insightsonindia.com/2016/02/01/in Conception and Pre-Natal Diagnostic
sights-daily-current-events-01-february-2016/ Techniques Act, 1994), which prohibits
ultrasonography for the purpose of determining
2. It is being said that the State Department of the the sex of the foetus.
United States of America (USA) is actively
considering a reversal of the policy of de- http://www.insightsonindia.com/2016/02/02/in
hyphenation (dealing with India and Pakistan in sights-daily-current-events-02-february-2016/
silos). What are some of the likely outcomes if
this proposal is carried through? 4. The Nodal Agency in India for investigation of
any Zika Virus outbreak is
1. India would be seen to be supporting USA
policy on Afghanistan a) National Institute of Virology (Pune)
2. India-Pakistan disputes will be resolved b) National Centre for Disease Control (Delhi)
Select the correct answer using the code given c) Centre for Disease Control and Prevention
below: d) National Vector Borne Disease Control
Programme
(a) 1 Only
(b) 2 Only Solution: b.
c) Both 1 and 2
d) Neither 1 nor 2 The Union Health Ministry of India has
sounded an alert for Zika and appointed the
Solution: a. National Centre for Disease Control (NCDC)
as the nodal agency for investigation of any
The US is now considering a proposal to re- outbreak of the viral infection in India.
merge the office of the Special Representative
for Afghanistan and Pakistan (SRAP) back http://www.insightsonindia.com/2016/02/03/in
with the Bureau of South and Central Asia sights-daily-current-events-03-february-2016/
(SCA) that handles India, the rest of the
subcontinent and Central Asian republics.
5. With regard to two new committees set up by
the Ministry of Finance Tax Policy Council China, India and Russia are the three largest
(TPC) and Tax Policy Research Unit (TPRU) shareholders.
which of the following statements about them
is NOT correct? Former chief secretary of Gujarat D J Pandian
has been appointed vice-president and chief
a) The decision to constitute these investment officer of the newly-created Asian
Committees is based on the recommendation of Infrastructure Investment Bank (AIIB).
the Tax Administration Reform Commission
b) The TPC will be headed by the Union http://www.insightsonindia.com/2016/02/04/in
Finance Minister sights-daily-current-events-04-february-2016/
c) The TPRU will be headed by the Revenue
Secretary 7. Indias National Manufacturing Policy (NMP),
d) Tax policy and related legislation was which is nearing finalisation, supports the
earlier handled by the Central Board of Direct application of Compulsory Licensing (CL)
Taxes and Central Board of Excise and across different manufacturing sectors. In this
Customs context, which of the following statements
about CL is/are incorrect?
Solution: c.
1. Access to a CL permits the CL-holder to
Tax Policy Research Unit (TPRU) shall manufacture a patented product without the
comprise of officers from both the Central patent owners consent
Board of Direct Taxes (CBDT) and Central 2. So far, India has issued only two CLs one
Board of Excise and Customs (CBEC) as well for the Tandem Organic Solar Cell
as economists, statisticians, operational Architecture by NanoFlex and another for
researchers, legal experts. The unit will be the anti-cancer medicine Nexavar by
headed by an officer of the level of Chief Bayer
Commissioner at functional level alternatively Select the correct answer using the code given
from CBDT and CBEC for a fixed tenure, who below:
will directly report to Revenue Secretary.
a) 1 Only
http://www.insightsonindia.com/2016/02/03/in b) 2 Only
sights-daily-current-events-03-february-2016/ c) Both 1 and 2
d) Neither 1 nor 2
http://pib.nic.in/newsite/PrintRelease.aspx?reli
d=136014 Solution: b.

6. With reference to the newly-created Asian So far, India has issued only one CL. In March
Infrastructure Investment Bank, which of the 2012, Natco Pharma was granted a license for
following statements is/are correct? an anti-cancer medicine Nexavar patented by
Bayer.
1. It was formally set up in Beijing in 2015
2. China, India and Germany are the three http://www.insightsonindia.com/2016/02/05/in
largest shareholders sights-daily-current-events-05-february-2016/
3. It will invest in, among other sectors, social
infrastructure 8. The Geographical Indications of Goods
Select the correct answer using the code given (Registration and Protection) Act,1999 is
below: administered by the

a) 1 Only a) Controller General of Export Accounts


b) 1 and 2 Only b) Controller General of Patents, Designs and
c) 1 and 3 only Trade Marks
d) 1, 2 and 3 c) Ministry of Culture
d) Controller General of Geographical
Solution: c. Wonders
Lack of political will
Solution: b. Despite lower overall health spending,
national spending on malaria control increased
In December 1999, the Parliament had passed from Rs.400 crore to Rs.500 crore last year, and
the Geographical Indications of Goods a new procurement policy has gone a long way
(Registration and Protection) Act,1999. This towards speeding up the purchase of nets and
Act seeks to provide for the registration and pesticides
better protection of geographical indications
relating to goods in India. The Act would be http://www.thehindu.com/opinion/op-ed/on-
administered by the Controller General of malaria-the-governments-rhetoric-must-meet-
Patents, Designs and Trade Marks- who is the reality/article8203628.ece
Registrar of Geographical Indications. The
Geographical Indications Registry would be 10. To prevent diet-related chronic diseases,
located at Chennai. micronutrient deficiencies and reduce the risk
of contracting Non-Communicable Diseases,
http://www.insightsonindia.com/2016/02/06/in the World Health Organisation recommends
sights-daily-current-events-06-february-2016/
a) a daily intake of at least 100 grams of fruits
9. In the context of efforts to control/eliminate and vegetables
malaria from our country, which of the b) a daily intake of at least 200 grams of fruits
following statements is/are incorrect? and vegetables
c) a weekly intake of at least 1200 grams of
1. The National Vector Borne Disease Control fruits and vegetables
Programme is the nodal agency for d) a daily intake of at least 400 grams of fruits
coordination of the malaria programme and vegetables
2. Currently, malaria control officials are
responsible for reporting their own progress Solution: d.
3. Successive budgets over the past few years
have allocated lesser funds for malaria WHO: Increasing fruit and vegetable
control programmes consumption to reduce the risk of non-
Select the correct answer using the code given communicable diseases
below:
Indians fall terribly short of the
a) 1 and 2 Only recommendation. This finding is part of the
b) 2 Only Indias Phytonutrient Report brought out by a
c) 1 and 3 Only Delhi-based think tank, the Indian Council for
d) 3 Only Research on International Economic Relations
(ICRIER), which conducted the study across
Solution: d. five cities covering 1,001 individuals from the
upper and middle income groups.
Some issues plaguing the malaria control
programme that are common to many other http://www.thehindu.com/opinion/op-
government programmes include: ed/apples-and-oranges-are-the-meat-of-the-
matter/article8203634.ece
Poor data collection/monitoring/evaluation
(Under-reporting, unreliable data) 11. Consider the following statements:
Understaffed clinics
Controversies in procurements (in this case, 1. The past two successive Union Budgets
procurement of mosquito nets using WB- have allocated lesser money to the health
funded money) sector
Shortage of medicines 2. Under the Union Health Ministrys Sarva
Administrative costs overshooting costs Swasthya Abhiyan, fifty essential
associated with the actual prevention and medicines are accessible to the public for
control of disease (medicines, nets, insecticide free in public health facilities
sprays etc) Which of the statements above is/are correct?
a) 1 Only
b) 2 Only
c) Both 1 and 2
d) Neither 1 nor 2

Solution: a.

Two successive Union Budgets have slashed


support to the health sector, and by more than
20 per cent. These cuts have brought
programme implementation in already under-
resourced areas to a virtual standstill.

There have been vague assurances that some


small incentives would be extended to States
that initiate free medicine schemes, a far cry
from the vision initially that promised Central
funding for all such schemes. Amid conflicting
statements, the only assurance that has held
ground is on support to the availability of just
50 medicines. The actual implementation of
even this grossly inadequate provision has not
been spelt out.

http://www.thehindu.com/opinion/op-ed/not-
a-good-prognosis/article8203614.ece
1. One of the new approaches for mosquito d) The northernmost point on the demarcated
eradication that show promise in the war LoC
against Zika and Dengue Virus is the use of
Genetically Engineered (GM) Mosquitoes. In Solution: d.
what way do these mosquitoes help?
Point NJ9842
a) Male GM Mosquitoes indirectly block
dengue transmission to humans, as their sperms https://en.wikipedia.org/wiki/NJ9842
kill the virus inside the body of the mosquitoes
they breed with 3. Consider the following statements:
b) Female GM mosquitoes pass on dominant
lethal genes which prevent the multiplication 1. Altitude sickness occurs at higher altitudes
of virus strains to their offspring and due to lower percentage of oxygen in the
gradually all female mosquitoes will contain atmosphere
this gene 2. Acclimatisation, or homeostasis, refers to
c) Tetracycline is a chemical compound any of the numerous gradual, long-term
which GM Mosquitoes produce. This responses of an organism to changes in its
compound prevents mosquitoes from environment
successfully reproducing, thus reducing Which of the statements above is/are correct?
population significantly in a few weeks
d) Male GM Mosquitoes pass on a dominant a) 1 Only
lethal gene to offspring which prevents b) 2 Only
mosquito larvae from growing properly and c) Both 1 and 2
causes them to die before adulthood d) Neither 1 nor 2

Solution: d. Solution: d.

Two novel strategies have shown promise in At sea level, because air is compressible, the
recent years. One involves the use of weight of all that air above us compresses the
genetically modified male Aedes aegypti air around us, making it denser. As you go up a
mosquitoes that carry a dominant lethal gene. mountain, the air becomes less compressed and
The gene is passed down to offspring when is therefore thinner. The important effect of this
they mate with wild female mosquitoes that are decrease in pressure is this: in a given volume
not genetically altered. The lethal gene in the of air, there are fewer molecules present. This
offspring produces a protein that stops their is really just another way of saying that the
cells from functioning normally by producing pressure is lower. The percentage of those
more of itself and prevents other genes molecules that are oxygen is exactly the same:
essential for survival from turning on. This 21%. The problem is that there are fewer
prevents the mosquito larvae from growing molecules of everything present, including
properly and causes them to die before oxygen. So although the percentage of oxygen
adulthood, essentially breaking the insects life in the atmosphere is the same, the thinner air
cycle. means there is less oxygen to breathe.

http://www.thehindu.com/news/national/with- Homeostasis: The numerous sudden changes


zika-indian-firm-scales-up-trials-for-gm- that evoke rapid and short-term responses via
mosquitoes/article8206480.ece the nervous and hormonal systems are not
examples of acclimatization (What is
2. Point NJ9842 refers to acclimatisation?). An individual organism can
regulate its internal processes rapidly to sustain
a) The southernmost point on the demarcated itself within the usual range of environmental
Line of Control (LoC) changes that it encounters hourly or daily. But
b) The northernmost point on the Siachen this rapid regulation, or homeostasis, is limited
Glacier in its operation to a small range of
c) The northernmost point on the demarcated environmental variations.
Line of Actual Control
http://www.thehindu.com/news/national/army since has been under an indefinite
s-unseen-enemy-in-siachen/article8206435.ece moratorium on its cultivation
Which of the statements above is/are correct?
4. With reference to the Civil Liabilities for
Nuclear Damage Act, 2010, which of the a) 1 Only
following statements is/are incorrect? b) 2 Only
c) Both 1 and 2
1. Section 17(b) permits victims of a nuclear d) Neither 1 nor 2
incident to sue the operator or the supplier
for damages applying tort law and thus Solution: c.
exposing suppliers to unlimited liability
2. Under Section 46 liability for a nuclear The Indian GM crops saga is a convoluted
accident can be channelled from the one. Currently, it has the worlds fourth largest
operator to suppliers of nuclear material GM crop acreage on the strength of Bt cotton,
Select the correct answer using the code given the only genetically modified crop allowed in
below: the country.

a) 1 Only Following the study of biosafety data and field


b) 2 Only trials by two expert committees, Bt brinjal was
c) Both 1 and 2 cleared for commercialization by Indias top
d) Neither 1 nor 2 biotech regulator, the Genetic Engineering
Appraisal Committee, in 2009. But nothing
Solution: c. came of it, with moratoriums imposed by then
Union environment minister Jairam Ramesh
Both are incorrect. Provisions of each section and his successor Jayanthi Natarajan following
have been interchanged in the statements opposition from civil society groups and
above. brinjal-growing states.

Under Section 17(b), liability for a nuclear http://www.livemint.com/Opinion/2KSuCjSTz


accident can be channelled from the operator, 3qvRG9OyqHBJM/Genetically-modified-
which is the Nuclear Power Corporation of crops-the-way-forward.html
India, to suppliers of nuclear material,
specifically if the accident is due to an act of the
supplier or his employee, which includes
supply of equipment or material with patent or
latent defects or sub-standard services.

Section 46 permits victims of a nuclear incident


to sue the operator or the supplier for damages
applying tort law, even though such
proceedings would be beyond the scope of
CLNDA and its liability cap, and thus exposing
suppliers to unlimited liability.

http://www.thehindu.com/opinion/editorial/nu
clear-ambiguities/article8206363.ece

5. Consider the following statements:

1. Bt cotton is the only genetically modified


crop allowed to be grown by farmers in
India
2. Bt brinjal was cleared for
commercialization by the Genetic
Engineering Appraisal Committee, but it
1. The recently launched Project Sunrise, The single directive provision (of the CVC
associated with the states of Northeast India, is Act) was struck down by the Supreme Court as
it discriminated between officers of and above
a) A programme by the Ministry of Health and the rank of Joint Secretary and the rest of the
Family Welfare that will complement the bureaucracy.
regions National AIDS Control Programme
b) A Start Up India, Stand Up India sub- Autonomy to anti-corruption agencies in
scheme for the region launched by the Ministry initiating a Preliminary Enquiry or regular case
of Commerce and Industry will not result in miscarriage of justice because
c) A Mission Mode Project initiated by the the ultimate power of sanction to prosecute a
Ministry of Railways to connect all 8 state government official in a court of law rests with
capitals by 2019 the government. What should give confidence
d) A Mission Mode Project initiated by the to both Central and a State government is the
Ministry of Power to generate surplus fact that the Supreme Court has not disputed
electricity in the region by 2019 their competence in granting or denying
sanction requested by an investigating agency.
Solution: a. The court has merely demanded greater speed
in a government decision in this regard.
http://www.insightsonindia.com/2016/02/08/in
sights-daily-current-events-08-february-2016/ http://www.thehindu.com/opinion/op-ed/an-
order-against-the-antigraft-
2. United Through Oceans is the theme for fight/article8210121.ece

a) Ministry of Shippings Project Sagarmala 4. The Mandal Commissions recommendations


b) The Joint Space Project of SAARC nations paved the way for reservation of government
c) The International Fleet Review 2016, jobs for the Other Backward Classes. Under
hosted by the Indian Navy which article of the Indian Constitution did the
d) The International Maritime Organisations President have the authority to set up this
World Maritime Day 2016 commission?

Solution: c. a) Article 15 (Prohibition of discrimination on


certain grounds)
https://ifr16.indiannavy.gov.in/logo-mascot- b) Article 16 (Equality of opportunity in
theme-ifr16-theme-song.htm public employment)
c) Article 340
3. Consider the following statements: d) Directive Principles of State Policy

1. The single directive provision (of the Solution: c.


CVC Act) was struck down by the Supreme
Court as it discriminated between common Article 340 in The Constitution Of India:
citizens and central government officials Appointment of a Commission to investigate
2. The ultimate power of sanction to prosecute the conditions of backward classes
a central/state government official in a
court of law rests with the central/state Chapter 7, Indian Polity 4th Edition by M
government Laxmikanth
Which of the above statements is/are true?
Improvisation:
a) 1 Only http://www.thehindu.com/opinion/op-ed/the-
b) 2 Only dalithindutva-paradox/article8210119.ece
c) Both 1 and 2
d) Neither 1 nor 2 5. Consider the following statements:

Solution: b. Assertion (A): The Siachen glacier has seen an


increase in erratic events (disasters) in recent
times
Reason (R): Shrinking glaciers affect the slope
gradient of mountains, which in turn affects
their stability

Select the correct answer:

a) A and R both are true, and R is the correct


explanation for A
b) A and R both are true, and R is the NOT the
correct explanation for A
c) A is correct, R is incorrect
d) A is incorrect, R is correct

Ans: a.

A small board at the Siachen Base Camp


reads: Snout of the glacier was here on 10
April 2005. Today the snout, the starting point
of the Siachen glacier, has moved about a
kilometre ahead from that point. It is a
testimony to the accelerating pace of human-
induced climate change and is the likely culprit
behind increased disasters on the glacier.

Dr. Tayal, who studies glaciers in Kashmir


and the North-East, said glaciers were not only
shrinking, but also shortening and thinning
because of which they affect the local
conditions. Glaciers are typically present
between mountains, and their shrinking affects
the slope gradient of mountains, which in turn
affects their stability, he observed. This
resulted in an increase in landslides and
avalanches. This is precisely what is unfolding
on Siachen, which sits between the Karakoram
ranges.

http://www.thehindu.com/news/national/army
s-unseen-enemy-in-siachen/article8206435.ece
1. With reference to India economy, consider the Page 41, Macroeconomics Class XII NCERT
following:
RBI buys Rs 10,000 crore via OMOs
1. Bank rate
2. Open market operations 3. The Ministry of Petroleum & Natural Gas
3. Public debt recently released the Hydrocarbon Vision
4. Public revenue 2030 for North-East India document, outlining
Which of the above is/are steps to leverage the hydrocarbon sector for
component/components of Monetary Policy? development of the region. The states NOT
covered by it include:
a) 1 Only
b) 2 and 4 Only 1. West Bengal
c) 1 and 2 only 2. Sikkim
d) 1, 3 and 4 only 3. Nagaland
4. Mizoram
Solution: c. Select the correct answer using the code given
below:
CS(P) 2015
a) 1 Only
RBI buys Rs 10,000 crore via OMOs b) 1 and 2 Only
c) 1,2 and 4 Only
2. Consider the following statements: d) 2,3 and 4 Only

1. When the Reserve Bank of India (RBI) sells Solution: a.


government securities to the general public
it decreases the stock of high powered The objectives of the plan are to leverage the
money in the economy regions hydrocarbon potential, enhance access
2. The total liability of the RBI is called high to clean fuels, improve availability of
powered money, which also consists of petroleum products, facilitate economic
currency held by the public development and to link common people to the
Which of the statements above is/are correct? economic activities in this sector. The states
covered include Arunachal Pradesh, Assam,
a) 1 Only Manipur, Meghalaya, Mizoram, Nagaland,
b) 2 Only Sikkim and Tripura.
c) Both 1 and 2
d) Neither 1 nor 2 http://pib.nic.in/newsite/PrintRelease.aspx?reli
d=136255
Solution: c.
4. India is said to be taking steps in order to be at
High Powered Money: The total liability of the the forefront of the war against Neglected
monetary authority of the country, RBI, is Tropical Diseases (NTDs). What are NTDs?
called the monetary base or high powered
money. It consists of currency (notes and coins a) a group of infectious diseases which
in circulation with the public and vault cash of primarily affect the poorest sections of society,
commercial banks) and deposits held by the in both rural and urban areas
Government of India and commercial banks b) a group of infectious diseases which
with RBI. If a member of the public produces a primarily affect the tribal population
currency note to RBI the latter must pay her c) a group of infectious diseases which affect
value equal to the figure printed on the note. people regardless of their socio-economic
Similarly, the deposits are also refundable by status and are characterized by little attention
RBI on demand from deposit-holders. These from policy-makers
items are claims which the general public, d) None of the above
government or banks have on RBI and hence
are considered to be the liability of RBI. Solution: a.
The neglected tropical diseases (NTDs) are a
group of infectious diseases which primarily Keeping in mind the national requirement for
affect the poorest sectors of society, especially augmenting milk production, development
the rural poor and the most disadvantaged programmes focussed on promoting cross
urban populations. Nearly one billion people in breeding of dairy cows. This resulted in neglect
the world suffer from NTDs, which are referred of our indigenous cattle breeds. It is time now
to as neglected because they are that programmes focussing on improvement in
characterized by little attention from policy- productivity of indigenous cattle is developed
makers, lack of priority within health strategies, and implemented intensively. A new initiative
inadequate research, limited resource as National Gokul Mission has been launched
allocation and few interventions. for the preservation and promotion of
indigenous breed of cows under national
The Union Health Minister stated that India bovine genetic and dairy development
shall be in the forefront of the war against programme for the first time in the country.
Neglected Tropical Diseases. Shri Nadda said:
the National Deworming Day will mobilize Enhancing milk production and productivity is
health personnel, state governments and other one of the objectives, not primary.
stakeholders to prioritize investment in control
of Soil Transmitted Helminth (STH) http://pib.nic.in/newsite/PrintRelease.aspx?reli
infectionsone of the most common d=136246
infections.
6. The nodal agency authorised to possess and
http://pib.nic.in/newsite/PrintRelease.aspx?reli carry out research on meteorites in India is
d=136254
a) Indian Institute of Astrophysics
5. Which of the following statements with b) Indian Space Research Organisation
reference to Indias dairy sector is/are c) Geological Survey of India
incorrect? d) National Institute of Meteor Science

1. India occupies the top position in the world Solution: c.


in terms of milk production
2. The primary objective of the National In the wake of two suspected meteorite falls in
Gokul Mission is to bridge the gap in milk Vellore district in the last fortnight, the
productivity between India and developed Geological Survey of India (GSI) has written to
dairy nations the State government seeking a direction to all
Select the correct answer using the code given District Collectors that any such cases be
below: reported to GSI and any finds handed over, as
it was the only nodal agency authorised to
a) 1 Only possess and carry out research on meteorites in
b) 2 Only the country.
c) Both 1 and 2
d) Neither 1 nor 2 http://www.thehindu.com/todays-paper/tp-
national/gsi-says-it-is-custodian-of-meteorite-
Solution: b. falls/article8215933.ece

In its journey to the top position in the world 7. Which of the following terms is/are associated
in terms of milk production, the sector has with water supply/sources from medieval
witnessed several structural changes in India?
production, processing and consumption that
have been conditioned by the changing socio- 1. Bawadis
economic conditions in the country. Today we 2. Karez
have been able to provide on an average 302 gm 3. Gunjs
per person per day milk in the country which is Select the correct answer using the code given
more than the minimum required below:
recommended by the WHO.
a) 1 Only
b) 2 and 3 Only
c) 1 and 3 Only
d) 1, 2 and 3

Solution: d.

We are conducting a preliminary survey of all


major water sources of Adil Shahi era, which
includes, Karez (meaning underground water
supply system in Persian), Gunjs (overhead
stone tanks), and Bawadis (open wells) of the
city. Within a couple of months, we expect to
submit the preliminary report to the
government, said Kishore Raykar, director of
the agency.

http://www.thehindu.com/todays-paper/tp-
national/ancient-water-supply-system-in-
karnataka-being-mapped/article8215934.ece

8. The implementation of the Foreign


Contribution (Regulation) Act is administered
by

a) Ministry of Home Affairs


b) Reserve Bank of India
c) Ministry of Finance
d) The Intelligence Bureau

Solution: a.

Unlike other international donors, the BMGF


is not registered under the Foreign Contribution
Regulation Act (FCRA), leaving its funding out
of the ambit of the Ministry of Home Affairs
(MHA), which is the regulatory authority for
NGOs and associations who receive foreign
funds

http://www.thehindu.com/news/national/gates-
foundation-on-centres-
radar/article8215060.ece

https://fcraonline.nic.in/home/index.aspx
1. To ensure energy security, the Government of Health & Family Welfare and Human Resource
India is setting up strategic crude oil storages at Development.
three locations. Which of the following are
among these locations? Project Implementation: The Ministry of
Women and Child Development would be
1. Vishakhapatnam, Andhra Pradesh responsible for budgetary control and
2. Padur, Kerala administration of the scheme from the Centre.
3. Mangalore, Karnataka At the State level, the Secretary, Department of
4. Ratnagiri, Maharashtra Women and Child Development will be
Select the correct answer using the code given responsible for overall direction and
below implementation of the scheme.

a) 1, 2 and 3 Only http://pib.nic.in/newsite/PrintRelease.aspx?reli


b) 2 and 4 Only d=136289
c) 1 and 3 only
d) 1, 2, 3 and 4 3. Consider the following statements:

Solution: c. 1. Pakistan views the Line of Control (LoC) as


going from Point NJ9842 along the Saltoro
India is building underground storages in Range to the Karakoram Pass
Visakhapatnam and Mangalore and Padur in 2. India on the other hand views the LoC as
Karnataka to store about 5.33 million tonnes of going straight from Point NJ9842 to the
crude oil to guard against global price shocks Karakoram Pass
and supply disruptions. Which of the statements above is/are correct?

These strategic storages would be in addition a) 1 Only


to the existing storages of crude oil and b) 2 Only
petroleum products with the oil companies and c) Both 1 and 2
would serve as a cushion in response to external d) Neither 1 nor 2
supply disruptions. The construction of the
strategic crude oil storage facilities is being Solution: d.
managed by Indian Strategic Petroleum
Reserves Limited (ISPRL), a Special Purpose This question was framed in order to learn
Vehicle, which is a wholly owned subsidiary of about the political geography of this area of
Oil Industry Development Board (OIDB). dispute.

http://www.thehindu.com/todays-paper/tp- Pak LoC


national/uaes-adnoc-to-store-crude-in-
india/article8220460.ece Pakistan argues that this means that the line
should go from NJ 9842 straight to the
2. Beti Bachao, Beti Padhao scheme is Karakoram pass on the Sino-Indian border.
administered by India, however, insists that the line should
proceed north from NJ 9842 along the Saltoro
a) Ministry of Women and Child range to the border with China. Between these
Development two interpretations lies a substantial amount of
b) Ministry of Health and Family Welfare glaciated territory that both sides want control
c) Ministry of Human Resource Development of.
d) a, b and c

Solution: a.

Beti Bachao Beti Padhao (BBBP) Scheme was


launched on 22nd January, 2015 at Panipat in
Haryana. It is a tri-ministerial effort of
Ministries of Women and Child Development,
http://pib.nic.in/newsite/PrintRelease.aspx?reli
d=136255

4. With reference to the Hydrocarbon Vision 2030


for North-East India, which of the following
statements is/are incorrect?

1. The vision statement lays out a detailed


roadmap for the upstream (Exploration and
Production) sector of the hydrocarbon
value chain only
2. The Vision aims at doubling Oil and Gas
production by 2030 and promoting
cooperation with neighbouring countries in
the hydrocarbon sector
Select the correct answer using the code given
below:

a) 1 Only
b) 2 Only
c) Both 1 and 2
d) Neither 1 nor 2

Solution: a.

Beyond production, the focus areas include


exploring hydrocarbon linkages and trade
opportunities with Bangladesh, Myanmar,
Nepal & Bhutan; implementation of Make In
India in the region; development of health &
medical facilities; industrial policy &
infrastructure related action points; focus on
skill development; and employment generation
requirement in the region. The vision statement
lays out a detailed roadmap for the entire
hydrocarbons value chain, covering upstream,
midstream and downstream segments. This
report includes an action plan of immediate,
medium-term and long-term initiatives to
help achieve the objectives.

The Vision aims at doubling Oil & Gas


production by 2030, making clean fuels
accessible, fast tracking projects, generating
employment opportunities and promoting
cooperation with neighbouring countries.
1. What is the significance of the ability to detect 1. Its recent detection for the first time was
gravitational waves, or as they are also made using the technique of Laser
known, space time ripples? Interferometry
2. These waves are actually space and time
1. We may be able to understand the events which are waving
that unfolded at the moment of the Big 3. Everything with mass and/or energy can
Bang make gravitational waves
2. We can study any phenomena which do not Select the correct answer using the code given
emit light or particles, but emit below:
gravitational waves
Select the correct answer using the code given a) 1 Only
below: b) 3 Only
c) 2 and 3 only
a) 1 Only d) None
b) 2 Only
c) Both 1 and 2 Solution: d.
d) Neither 1 nor 2
Just how small is the effect?
Solution: c.
The effect of GWs is very weak, however, and
So we have a new way to probe black holes? only the biggest masses, moving under the
greatest accelerations, are expected to warp
Not just black holes, but the dark Universe in their surroundings to any appreciable degree.
general. So much of what we theorise to be out The Advanced Laser Interferometry
there does not radiate light in any of its forms Gravitational-Wave Observatory (LIGO)
from gamma-rays to the ultraviolet, from the interferometers have been searching for this
visible to radio waves or emit particles. And stretching and squeezing for over a decade,
unlike light or particles, gravitational waves gradually improving the sensitivity of their
cannot be blocked or deflected; they will pass equipment. The expectation was that their
through any and all obstructions unhindered. experiments would need to detect disturbances
And that makes them a free pass to begin no bigger than a fraction of the width of a
exploring phenomena that were previously off proton.
limits. We know, for example, that it is
impossible to see across space to before Everything with mass and/or energy can make
380,000 years after the Big Bang the Universe gravitational waves, but much of it is
hadnt cooled sufficiently until that point to practically undetectable. Watch the video, apart
permit light to propagate. But, theoretically, from going through the article, provided on the
there should still be background gravitational following webpage:
waves washing over us from the very earliest
moments of the expanding cosmos. If future http://www.thehindu.com/sci-
space-borne gravitational-wave observatories tech/science/gravitational-waves-landmark-
can detect this remnant signal, it will bring us announcement-may-shed-light-on-early-
closer than ever to understanding what universe/article8223840.ece
happened at T=0.
3. The World Banks recently released World
http://www.thehindu.com/sci- Development Report (WDR) Digital
tech/science/gravitational-waves-landmark- Dividends finds that
announcement-may-shed-light-on-early-
universe/article8223840.ece a) Digital technologies have spread rapidly
throughout much of the world and the
2. Which of the following statements associated associated digital dividends have been
with Gravitational Waves is/are incorrect? commensurate
b) Digital technologies have spread rapidly
throughout much of the world, but their digital
dividends have lagged behind
c) Closing or reducing the digital divide people who could access the internet (and of
(access to the Internet) has shown to always course, only a small part of it).
yield commensurate dividends
d) Increased adoption of digital technologies But greater digital adoption will not be
has resulted in an unintended consequence - enough. To get the most out of the digital
increased transparency of governments revolution, countries also need to work on its
analogue complements by strengthening
Solution: b. regulations that ensure competition among
businesses, by adapting workers skills to the
Option a and c are similar. Both are demands of the new economy, and by ensuring
incorrect. Option d may be true, but it is often that government institutions and others are
a choice that the governments make whether accountable.
or not to leverage technology to increase
transparency. The core finding of the report, is The slow pace of improvement of the quality
option b. of basic infrastructure expressways,
logistics, storage, postal delivery system and
The WDR finds that digital technologies have reliable supply of electricity have also
spread rapidly throughout much of the world, hampered the growth of e-commerce in India.
but their digital dividends the broader Hampered growth of e-commerce implies
development benefits from using these that we have been unable to fully harness the
technologies have lagged behind. In many potential of digital technologies. Hence, good
instances digital technologies have boosted quality basic infrastructure too is a part of
growth, expanded opportunities, and improved analogue complements.
service delivery. Yet their aggregate impact has
fallen short and is unevenly distributed. http://www.thehindu.com/opinion/op-
ed/indias-digital-
http://www.thehindu.com/opinion/op- transformation/article8224206.ece
ed/indias-digital-
transformation/article8224206.ece 5. India occupied the Siachen Glacier in 1984 in a
preemptive move. What was this mission
4. The World Development Report (WDR) called?
Digital Dividends says that to get the most out
of a digital revolution, countries also need to a) Operation Snow
work on its analogue complements. Which of b) Operation Steeplechase
the following may NOT be regarded as one of c) Operation Meghdoot
these complements? d) Operation Rose

a) Encouraging innovative ideas such as Zero Solution: c.


Ratings
b) Enhancing workers skills The 76-km-long glacier in the Saltoro ridge
c) Government institutions should make beyond Point NJ 9842, where the Line of
themselves accountable Control ends, was left un-demarcated all these
d) Improving the quality of basic years. Pakistan began allowing tourists in the
infrastructure such as postal delivery systems 1970s, and there were occasional reports of its
and expressways attempt to occupy the glacier. In a pre-emptive
move, the Indian Army occupied the highest
Solution: a. peaks, at 21,000 feet, in April 1984 under
Operation Meghdoot. For 30 years, it remains
This question would have given you a hint for the longest continuing military operation
the answer to the third question! anywhere in the world.

Innovative ideas such as Zero Ratings would http://www.thehindu.com/news/national/he-


not be one of the analogue complements. Zero made-the-supreme-sacrifice-says-
Ratings would only help increase the number of pranab/article8224371.ece
1. Consider the following statements: http://pib.nic.in/newsite/PrintRelease.as
px?relid=136371
1. Agriculture and allied activities
employ about 49% of Indias total 3. The Ganga Sanskriti Yatra being
workforce organized by the Ministry of Culture
2. Agriculture sector accounts for only will showcase the culture heritage of
about 14% of Indias Gross Domestic River Ganga. Which of the following
Product statements about River Ganga is/are
Which of the statements above is/are incorrect?
correct?
1. Geologists believe that a few million
a) 1 Only years ago River Indus, Ganga and a
b) 2 Only part of Brahmaputra flowed as a
c) Both 1 and 2 single River Shiwalik
d) Neither 1 nor 2 2. In order from east to west, its left
bank tributaries include River
Solution: c. Gandak, Ghaghara and Gomati and
Ramganga
http://pib.nic.in/newsite/PrintRelease.as Select the correct answer using the code
px?relid=136375 given below:

2. With reference to the Consumer Price a) 1 Only


Index (CPI) released periodically by the b) 2 Only
Central Statistics Office (CSO), c) Both 1 and 2
Ministry of Statistics and Programme d) Neither 1 nor 2
Implementation, which of the following
statements is/are correct? Solution: d.

1. The base year of the CPI is taken as Both statements are correct.
2010
2. Separate CPI data is available for Geologists believe that a mighty river
rural and urban areas called Shiwalik or Indo-Brahma
3. CPI data is also available for each traversed the entire longitudinal extent
state of the Himalaya from Assam to Punjab
Select the correct answer using the code and onwards to Sind, and finally
given below: discharged into the Gulf of Sind near
lower Punjab, some 5-24 million years
a) 1 Only ago.
b) 2 and 3 Only
c) 1 and 3 only The important left bank tributaries (from
d) 1,2 and 3 west to east) are the Ramganga, the
Gomati, the Ghaghara, the Gandak, the
Solution: b. Kosi and the Mahananda.

The base year of the CPI is taken as Page 24-25, India Physical Environment
2012. Class XI NCERT
http://pib.nic.in/newsite/PrintRelease.as
px?relid=136349

4. Which of the following committees on


centre-state relations have dealt with
Article 356?

1. Sarkaria Commission
2. P V Rajamannar Committee
3. Punchhi Commission
4. S R Bommai Commission
Select the correct answer using the code
given below:

a) 1,2 and 3 Only


b) 2,3 and 4 Only
c) 1,2 and 4 Only
d) 1,2,3 and 4

Solution: a.

There was no Bommai Commission on


centre-state relations. But the S R
Bommai v/s Union of India case is the
landmark case associated with
Presidents rule.

Page 14.12 Indian Polity 4th Edition by


M Laxmikanth

http://www.thehindu.com/opinion/lead/t
he-presidential-
prerogative/article8229394.ece
1. Which among the following statements about c) Both 1 and 2
the Dam Rehabilitation and Improvement d) Neither 1 nor 2
Project is correct?
Solution: c.
a) Its objective is to improve the safety and
operational performance of selected existing An aquifer is an underground layer of water-
dams bearing rock, from which groundwater can be
b) Its objective is to improve the outcome of extracted. The mapping would assist in
efforts towards rehabilitation of displaced estimating the quantity and quality of ground
communities due to dam construction water in an aquifer and would help in
c) Project implementation costs will be shared assessment of sustainable level of ground water
by the central and respective state government extraction. This would also help in making the
d) It is a project initiated by the South Asia Country climate change resilient.
Network on Dams, Rivers and People
(SANDRP) in the South Asian region to create This mapping is being undertaken by Central
awareness among affected communities about Ground Water Board (CGWB). The final
their rights objective of the programme is to prepare
Management Plans, in consultation with
Solution: a. stakeholders and State Governments, which
will identify the recharge and other measures to
In April 2012, the Central Water Commission replenish the declining trend of groundwater.
(CWC) with assistance from the World Bank, This Plan would help in predicting the ground
embarked upon the six year Dam Rehabilitation water scenario, the recharge potential and
and Improvement Project (DRIP) at an possibilities and would also help in educating
estimated cost of Rs. 2100 crore the stakeholders in water use efficiency.

The objective of the Dam Rehabilitation and http://pib.nic.in/newsite/PrintRelease.aspx?reli


Improvement Project for India is to improve the d=136334
safety and operational performance of selected
existing dams in the territory of the 3. With reference to the Himalayan Griffon or
participating states. The project development the Himalayan Vulture, which of the
objective (PDO) will be achieved through following statements is/are incorrect?
rehabilitation and improvement of dams and
improvement in central and state-level 1. The IUCN has listed it as a critically
institutional capacity to sustainably manage endangered species
dam safety administration and operation and 2. Apart from the Himalayas, this bird can be
maintenance. found in South India as well as Central Asia
Select the correct answer using the code given
http://www.insightsonindia.com/2016/02/13/in below:
sights-daily-current-events-13-february-2016/
a) 1 Only
2. The Ministry of Water Resources is b) 2 Only
undertaking an Aquifer Mapping (AM) c) Both 1 and 2
project. Which of the following statements d) Neither 1 nor 2
associated with AM is/are correct?
Solution: a.
1. It would assist in estimating the quantity as
well as quality of ground water in an aquifer The species has been listed as Near Threatened
2. It can help make India climate change by the IUCN. In recent years the bird has been
resilient spotted in southern states including Kerala,
Select the correct answer using the code given Karnataka and Andhra Pradesh. It is also found
below: in western China, Kazakhstan, Uzbekistan,
Kyrgyzstan, Tajikistan, Afghanistan and
a) 1 Only Pakistan.
b) 2 Only
http://www.insightsonindia.com/2016/02/13/in b) The precipitation on land that does not run
sights-daily-current-events-13-february-2016/ off or recharge the groundwater but is stored in
the soil or temporarily stays on top of the soil
4. With reference to the chemical Pyriproxyfen, c) The relatively clean waste water from
consider the following statements: baths, sinks, washing machines, and other such
home appliances
Assertion (A): The World Health Organisation d) Ocean Water
has recommended its introduction in drinking
water supply in disease-carrying mosquito- Solution: a.
infested areas
There are specific geographical areas where the
Reason (R): It is a larvicide which produces water crisis is particularly acute. For instance,
malformations in mosquito larvae (it inhibits in the Ganges basin, blue water or fresh
the development of adult insect characteristics) water (surface water and aquifers) is being
consumed in a countercyclical fashion, with
Select the correct answer: water consumption being highest when water
availability is lowest.
a) A and R both are true, and R is the correct
explanation for A Researchers looked at blue water scarcity on a
b) A and R both are true, and R is the NOT the per-month basis dividing up the area according
correct explanation for A to specific grid cells. Water scarcity was
c) A is incorrect, R is correct calculated as the ratio of the blue water
d) A and R both are incorrect footprint (consumption) in a grid cell to the
total blue water availability in the cell.
Ans: c.
http://www.thehindu.com/opinion/op-
With direct causality between Zika virus and ed/inching-towards-a-crisis-drop-by-
microcephaly, a congenital brain development drop/article8234565.ece
anomaly, yet to be established, Latin American
doctors have come up with an alternative 6. As per the Agreement on the Monetary Policy
explanation: that a pesticide, Pyriproxyfen, was Framework between the Government of India
introduced into the drinking water supply in and Reserve Bank of India, which of the
2014. following statements is/are incorrect?

Pyriproxyfen is used in a state-controlled 1. A consumer inflation target of 4 per cent,


programme aimed at eradicating disease- with a band of plus or minus 2 percentage
carrying mosquitoes. The PCST report states, points, by the financial year ending in
Pyriproxyfen is a growth inhibitor of mosquito March 2017 has been set
larvae, which alters the development process 2. The central bank will be deemed to have
from larva to pupa to adult, thus generating missed its target if consumer inflation is at
malformations in developing mosquitoes and more than 6 percent or at less than 2 percent
killing or disabling them. It acts as an insect for three consecutive months
juvenile hormone or juvenoid, and has the Select the correct answer using the code given
effect of inhibiting the development of adult below:
insect characteristics (for example, wings and
mature external genitalia) and reproductive a) 1 Only
development. b) 2 Only
c) Both 1 and 2
http://www.thehindu.com/opinion/op-ed/is- d) Neither 1 nor 2
the-zika-threat-overhyped/article8234567.ece
Solution: b.
5. The term Blue Water refers to
As per the Agreement on the Monetary Policy
a) Fresh surface and groundwater Framework between the Government of India
and Reserve Bank of India, the two sides set a
consumer inflation target of 4 per cent, with a
band of plus or minus 2 percentage points, by
the financial year ending in March 2017. (But
the RBI has set on its own, a target of 5% by
March 17)

The central bank will be deemed to have missed


its target if consumer inflation is at more than 6
percent or at less than 2 percent for three
consecutive quarters (not months).

http://www.thehindu.com/todays-paper/tp-
business/reserve-bank-cant-drop-guard-on-
inflation/article8235185.ece

http://finmin.nic.in/reports/MPFAgreement28
022015.pdf
1. With regard to the Indias first mission to study 1. There was an active womens movement
the Sun, which of the following statements before 1947, which became dormant after
is/are correct? 2. The growth of what is termed as the
autonomous womens movements is called
1. Suryoday-1 will be placed in such an orbit by some as the second phase of the Indian
that it will be able to continuously view the womens movement
Sun without any eclipses Select the correct answer using the code given
2. This mission is meant to observe only the below:
solar corona it will carry one payload for
this purpose a) 1 Only
Select the correct answer using the code given b) 2 Only
below: c) Both 1 and 2
d) Neither 1 nor 2
a) 1 Only
b) 2 Only Solution: d.
c) Both 1 and 2
d) Neither 1 nor 2 An issue that is often raised is that if there was an
active womens movement before 1947, whatever
Solution: d. happened afterwards.

Aditya-L1, the Indian Sun mission due after In the mid 1970s there was a renewal of the
three years, may turn out to be a unique womens movement in India. Some call it the
formation of not one but two spacecraft looking second phase of the Indian womens movement.
at the Sun from two stable orbital points. That While many of the concerns remained the same
is if the Advisory Committee on Space there were changes both in terms of organisational
Sciences (ADCOS), which is the brain behind strategy as well as ideologies. There was the
the countrys extra-planetary missions, has its growth of what is termed as the autonomous
way. womens movements. The term autonomy
referred to the fact that they were autonomous or
Aditya is the nations third big extra-terrestrial independent from political parties as distinct from
outing after Moon and Mars, all conceived and those womens organisations that had links with
designed by ADCOS, the multi-faculty body of political parties.
the Indian Space Research Organisation.
Page 157-158, Social Change and development in
The Aditya-1 mission was initially conceived India Class XII NCERT
as a 400kg class satellite carrying one payload,
the Visible Emission Line Coronagraph http://www.thehindu.com/opinion/op-ed/a-
(VELC). the Aditya-1 mission has now been judgment-for-womens-rights/article8237561.ece
revised to Aditya-L1 mission and will carry
additional six payloads with enhanced science 3. Consider the following statements:
scope and objectives.
1. Swami Dayanand Saraswati supported
http://www.thehindu.com/todays-paper/tp- widow remarriage
national/aditya-gets-ready-to-gaze-at-the- 2. Schools for girls were established by the
sun/article8212387.ece Arya Samaj
3. The Arya Samaj sought to revive Vedic
2. A crucial point of the womens movement learning as modern education was detested
resonated in a recent Supreme Court order by the Samaj
emphasising that real development is only Which of the statements above is/are correct?
possible with womens economic
empowerment. Which of the following a) 1 Only
statements with reference to womens b) 1 and 2 only
movements in India is/are incorrect? c) 3 Only
d) 1, 2 and 3
Solution: b.
Arya Samaj: A North Indian Hindu reform http://www.thehindu.com/sci-tech/energy-and-
organisation of the late nineteenth and early environment/antarctica-influencing-weather-
twentieth centuries, particularly active in the in-tropics/article8233664.ece
Punjab, which sought to revive Vedic learning
and combine it with modern education in the 5. When corals are under stress, they expel the
sciences. algae living in their tissues causing the coral to
turn completely white. This is called coral
Page 383 Themes in Indian History III Class bleaching. In this context, which of the
XII NCERT following statements is/are correct?

Chapter 9 Our Pasts III Part 2 Class VIII 1. Corals can survive a bleaching event
NCERT 2. Cold water temperatures can result in coral
bleaching
http://pib.nic.in/newsite/PrintRelease.aspx?reli 3. Corals and algae live in a symbiotic
d=136393 relationship
Select the correct answer using the code given
4. Wind circulations from the surface upwards below:
and vice-versa are called cells, which set the
pattern for the general circulation of the a) 1 and 3 Only
atmosphere. Which of the following statements b) 2 and 3 Only
is/are incorrect? c) 1 and 2 Only
d) 1,2 and 3
1. The Hadley Cell is associated with the
tropics whereas the Ferrel Cell is associated Solution: d.
with the mid-latitudes
2. Changes in temperature of the polar region All statements are correct.
affects the strength of wind circulation in
the tropical cell Symbiotic associations are mutually useful
Select the correct answer using the code given associations. Ex: Algae and Fungi in Lichens.
below:
The coral provides the algae with a protected
a) 1 Only environment and the compounds necessary for
b) 2 Only photosynthesis, which are metabolic waste
c) Both 1 and 2 products of the coral. In return, the algae
d) Neither 1 nor 2 produce oxygen and help the coral to remove
wastes. Most importantly, they supply the coral
Solution: d. with organic products of photosynthesis which
are predominantly carbohydrates.
The temperature gradient between the equator
and the poles essentially drives the atmospheric When a coral bleaches, it is not dead. Corals
circulation in the southern hemisphere in the can survive a bleaching event, but they are
form of three north-south systems: the polar under more stress and are subject to mortality.
cell, the mid-latitude Ferrel cell and the tropical
Hadley cell. These cells are dynamically linked Not all bleaching events are due to warm
together. water. In January 2010, cold water
temperatures in the Florida Keys caused a coral
When the polar region warms, the location of bleaching event that resulted in some coral
the boundary between the polar and Ferrel cells death.
will change, along with the strength of http://oceanservice.noaa.gov/facts/coral_bleac
circulation in both cells. This in turn will h.html
influence the strength of tropical circulation on
the other side of the Ferrel cell. These linkages http://www.thehindu.com/sci-
between polar regions and mid- and tropical tech/science/unravelling-mysteries-of-coral-
latitudes are known as teleconnections. bleaching/article8233738.ece
1. Which of the following statements associated
with John Maynard Keynes is/are correct? Solution: d.

1. He tried to theorise about and explain the The Horn of Africa nation is strategically
fact that the economy may have long lasting located on the junction of the Indian Ocean and
unemployment the Red Seaa gateway to the Suez Canal via
2. He advocated that government fiscal policy the strait of Bab Al-Mandab.
should be used to stabilise the level of
output and employment http://www.thehindu.com/news/international/d
Select the correct answer using the code given jibouti-myanmar-and-sri-lanka-anchor-chinas-
below: maritime-silk-road/article8240828.ece

a) 1 Only 3. Consider the following State Associated Folk


b) 2 Only Dance pairs:
c) Both 1 and 2
d) Neither 1 nor 2 1. Arunachal Pradesh Bro-Zai
2. Assam Bardoishikla
Solution: c. 3. Mizoram Cheraw
4. Sikkim Hozagiri
The dominant thinking in economics before Which of the above pairs is/are correctly
Keynes was that all the labourers who are ready matched?
to work will find employment and all the
factories will be working at their full capacity. a) 1,2 and 3
This school of thought is known as the classical b) 2 and 3 Only
tradition. However, the Great Depression of c) 1 and 2 Only
1929 and the subsequent years saw the output d) 1 and 3 Only
and employment levels in the countries of
Europe and North America fall by huge Solution: a.
amounts. The fact that the economy may have
long lasting unemployment had to be theorised Tripura Hozagiri Dance
about and explained. Keynes book was an
attempt in this direction. Unlike his Destination North East 2016 Flyer
predecessors, his approach was to examine the
working of the economy in its entirety and http://pib.nic.in/newsite/PrintRelease.aspx?reli
examine the interdependence of the different d=136256
sectors. The subject of macroeconomics was
born. 4. Consider the following statements:

Page 4, 66 Macroeconomics Class XII NCERT 1. Wholesale prices as measured by the


Wholesale Price Index have been
Yet, back on his table again this year is continuously declining for over the past
Keynesian advice: invest your way out of sub- year
8 per cent GDP growth to steer the economy to 2. The continuous decline of overall
a higher trajectory. merchandise exports for over the past year
has been stemmed due to a sudden
http://www.thehindu.com/opinion/columns/ne expansion in shipments to the United States
ver-waste-a-crisis/article8241269.ece of America
Which of the statements above is/are correct?
2. The strait of Bab-el-Mandeb connects
a) 1 Only
a) Indian Ocean and Persian Gulf b) 2 only
b) North Atlantic Ocean and Mediterranean c) Both 1 and 2
Sea d) Neither 1 nor 2
c) Black Sea and the Mediterranean Sea
d) None of the above Solution: a.
Exports shrink for 14th straight month in http://pib.nic.in/newsite/PrintRelease.aspx?reli
January d=136406

Wholesale prices decline for 15th consecutive


month

5. Which among the following constitute Non-


Tax Revenue Receipts?

1. RTI Application fees


2. Spectrum charges
3. Dividends from the Reserve Bank of India
Select the correct answer using the code given
below:

a) 1 and 3 Only
b) 2 and 3 Only
c) 1 and 2 Only
d) 1,2 and 3

Solution: d.

The annual collection of Non Tax Receipts is


over Rs. 2 lakh crores. The biggest share flows
from Dividends paid by Public Sector
Undertakings, RBI etc. The other major items
of Non Tax Receipts are interest receipts,
spectrum charges, royalty, license fee, sale of
forms, RTI application fee etc.

http://pib.nic.in/newsite/PrintRelease.aspx?reli
d=136436

6. Following are commodity groups in the


Wholesale Price Index:

1. Primary Articles
2. Fuel and Power
3. Manufactured Products
Select the correct answer using the code given
below, where comparison is being made on the
basis of weights that have been accorded to
each of them:

a) 3>2>1
b) 3>1>2
c) 1>3>2
d) 2>1>3

Solution: b.

Manufactured Products (Weight 64.97%);


Primary Articles (Weight 20.12%); Fuel &
Power (Weight 14.91%)
1. India is said to be preparing to expand its
sphere of operations in the Indian Ocean a) 1 Only
Region (IOR). In this context, which of b) 2 Only
the following pairs is/are correctly c) Both 1 and 2
matched? d) Neither 1 nor 2

(Joint Solution: d.
(Participating
Military
country)
Exercise) Both statements are true.

1. Sri Lanka Ekuverin Tantia Tope, also spelled Tatya Tope or


Tantia Topi, (original name Ramchandra
Mitra Panduranga, born c. 181319, Pune,
2. Maldives
Shakti Indiadied April 18, 1859, Shivpuri),
Garuda was a leader of the Indian Mutiny of
3. Seychelles
Shakti 185758.
4. Indonesia Ex Limtaye
Tantia Tope was a Maratha Brahman in
Select the correct answer using the code the service of the former peshwa (ruler)
given below: of the Maratha confederacy, Baji Rao
(Peshwa Baji Rao II), and of his adopted
a) 1 and 3 Only son Nana Sahib, who was also
b) 2 and 3 Only prominent in the mutiny.
c) 1 and 2 Only
d) None http://pib.nic.in/newsite/PrintRelease.as
px?relid=136456
Solution: d.
3. The regulatory authority of the Atomic
Exercise with Sri Lanka Mitra Shakti Energy Regulatory Board (AERB) is
Exercise with Maldives Ekuverin derived from the rules and notifications
Exercise with Seychelles Ex Limtaye promulgated under the
Exercise with Indonesia Garuda Shakti
a) Atomic Energy Act, 1962
Exercise with Nepal Suryakiran b) Environment (Protection) Act, 1986
Exercise with China, Pakistan and c) Safety and Radiation Protection Act,
Thailand Cobra Gold (hosted by 1971
Thailand) d) Both a and b

2. Which of the following statements about Solution: d.


Ramachandra Panduranga is/are false?
An independent body, the Atomic
1. He was a Maratha Brahmin in the Energy Regulatory Board (AERB)
service of Peshwa Baji Rao II monitors safety. The safety standards
2. He was one of the rebel leaders in formulated by AREB are at par with
Indias first war of Independence those recommended by the international
Select the correct answer using the code bodies such as the International Atomic
given below: Energy Agency (IAEA) and the
International Commission on
Radiological Protection (ICRP). The http://pib.nic.in/newsite/PrintRelease.as
mission of the Board is to ensure that the px?relid=136450
use of ionizing radiation and nuclear
energy in India does not cause undue 5. Consider the following statements:
risk to health and the environment.
Assertion (A): There is still
The Atomic Energy Regulatory Board considerable resistance on the part of
was constituted on November 15, 1983 litigants in India to opt for mediation in
by the President of India to carry out the form of Alternate Dispute Resolution
certain regulatory and safety functions (ADR) methods
under the Act. The regulatory authority
of AERB is derived from the rules and Reason (R): The mediation process in
notifications promulgated under the India lacks proper statutory framework
Atomic Energy Act, 1962 and the
Environmental (Protection) Act, 1986. Select the correct answer:

Dr Jitendra Singh said, the safeguards a) A and R both are true, and R is the
for the purpose are observed as per the NOT the correct explanation for A
rules, procedures and principles laid b) A and R both are true, and R is the
down by the Atomic Energy Regulatory correct explanation for A
Board (AERB) under the DAE are c) A and R both are incorrect
among the best practices in the world. d) A is incorrect, R is correct

4. Albendazole is Ans: b.

a) An anthelminthic The Minister said ADR methods such


b) An Ethanol derivative which as Lok Adalats and Arbitration have
increases the Octane number of petrol gained considerable traction as an
c) Bharat Biotechs inactivated Zika effective means of dispute resolution.
Virus vaccine However, there is still considerable
d) None of the above resistance on the part of litigants to opt
for mediation due to the lack of proper
Solution: a. statutory framework for the mediation
process. It is in this regard the third
There have been a few reports in media agenda item explored the pre-litigation
that this medicine under the National dispute resolution scenario in India and
Deworming Programme is responsible the need to give statutory backing to
for complications in children. mediation by enacting a standalone
law.
Soil-transmitted helminth infections are
among the most common infections
worldwide and affect the poorest and
most deprived communities. The
National Deworming Programme seeks
to rid children of worms that cause these
infections.
1. With reference to the Government of 2. Paracel Islands
Indias decision to cut interest rates on 3. Spratly Islands
short-tenure small savings schemes, 4. Brunei
which of the following statements Select the correct answer using the
is/are correct? code given below:

1. This move could result in the a) 4-2-1-3


improvement of investor sentiment b) 3-2-4-1
2. The rates on such small savings c) 3-4-1-2
schemes would from now onward be d) 4-3-2-1
revised every quarter
3. High interest rates on small savings Ans: d.
schemes was one of the key reasons
for poor monetary policy http://www.thehindu.com/news/inter
transmission in recent times in the national/china-deploys-missiles-in-
economy disputed-south-china-
Select the correct answer using the sea/article8248382.ece
code given below:
3. The Union Cabinet recently gave its in
a) 1 and 2 Only principle approval to Project INDIGO.
b) 2 and 3 Only This project is being piloted by
c) 1 and 3 Only
d) 1,2 and 3 a) Department of Atomic Energy
b) Department of Science and
Solution: d. Technology
c) Indian Institute of Science
The finance ministry has also d) Both a and b
announced that the rates on such small
savings schemes would, henceforth, be Solution: d.
revised every quarter.
The proposal, known as LIGO-India
http://indianexpress.com/article/opini project (Laser Interferometer
on/editorials/kvp-finance-ministry- Gravitational-wave Observatory in
small-savings-no-small-gain/ India) is piloted by Department of
Atomic Energy and Department of
http://www.thehindu.com/opinion/edi Science and Technology (DST). The
torial/right-step-on-savings- LIGO-India project will establish a state-
schemes/article8249371.ece of-the-art gravitational wave
observatory in India in collaboration
2. What is the correct sequence of with the LIGO Laboratory in the U.S. run
occurrence of the following regions in by Caltech and MIT.
East Asia as one proceeds from South to
North? http://www.thehindu.com/news/natio
nal/indian-gravity-wave-detector-8-
1. Hong Kong years-away/article8238176.ece
d) Indian Institute of Himalayan
http://pib.nic.in/newsite/PrintRelease. Geology
aspx?relid=136479
Solution: b.
4. The Ministry of AYUSH has the mandate
to promote, propagate and globalize Khanij = minerals; Khoj = search
recognized Traditional and
Complementary Systems of Medicine The Operation Khanij Khoj project of
(T&CM) which include GSI (Ministry of Mines) launched during
the meeting is targeting concealed and
1. Naturopathy deep seated mineral deposits. GSI has
2. Unani planned two projects in (i) Northern
3. Siddha parts of Arravallies and Bundelkhand
4. Sowa-Rigpa craton; (ii) Western and Eastern
5. Homeopathy Dharwad cratons. It is focused on
Select the correct answer using the probing for deep seated/concealed
code given below: mineral deposits.

a) 2, 3 and 5 only
b) 1, 4 and 5 only
c) 1, 2, 3, 4 and 5
d) 1, 2, 3 and 5 only

Solution: c.

Ministry of AYUSH having the mandate


to promote, propagate and globalize
the recognized Traditional and
Complementary Systems of Medicine
(T&CM) including Ayurveda, Yoga,
Naturopathy, Unani, Siddha, Sowa
Rigpa and Homeopathy, proposes to
collaborate and cooperate with the
World Health Organization.

http://pib.nic.in/newsite/PrintRelease.
aspx?relid=136482

5. Operation Khanij Khoj is being


undertaken by

a) Ministry of Science and Technology


b) Ministry of Mines
c) Ministry of Earth Sciences
1. Which of the following statements about Spiti (Himanchal Pardesh) and Ladakh
Sowa-Rigpa, a recognized Traditional region of Jammu& Kashmir etc.
and Complementary System of
Medicine in India, are correct? http://pib.nic.in/newsite/PrintRelease.as
px?relid=136482
1. It is commonly known as Amchi
system of medicine 2. Which of the following statements about
2. Much of its theory and practice is the Bar Council of India is/are correct?
similar to Ayurveda
3. It is practiced in some parts of Nepal, 1. It is a statutory body
Bhutan and also in former Soviet 2. It prescribes standards of
Union professional conduct and etiquette
4. This medical tradition originated in for members of the bar
Tibet 3. It provides financial assistance to
Select the correct answer using the code organizes welfare schemes for the
given below: members
Select the correct answer using the code
a) 1,2 and 3 given below:
b) 1 and 3 Only
c) 2 and 3 Only a) 1 and 3 Only
d) 3 and 4 only b) 2 and 3 Only
c) 1, 2 and 3
Solution: a. d) 1 and 2 Only

Its origin remains uncertain. Solution: c.

Sowa-Rigpa commonly known as All statements are correct.


Amchi system of medicine is one of the
oldest, living and well documented The Bar Council of India (BCI) on
medical tradition of the world. It has Thursday told the Supreme Court that
been popularly practiced in Tibet, serious action would be taken against
Magnolia, Bhutan, some parts of China, the errant lawyers who brazenly defied
Nepal, Himalayan regions of India and the courts order for calm and indulged
few parts of former Soviet Union etc. in violence in the Patiala House courts
There are various schools of thought complex.
about the origin of this medical tradition,
some scholars believe that it is The Bar Council of India (BCI) is a
originated from India, some says China statutory body created by Parliament to
and others consider it to be originated regulate and represent the Indian bar (It
from Tibet itself. The majority of theory was established under the Advocates
and practice of Sowa-Rigpa is similar to Act, 1961). We perform the regulatory
Ayurveda. function by prescribing standards of
professional conduct and etiquette and
In India this system has been practiced by exercising disciplinary jurisdiction
in Sikkim, Arunachal Pardesh, over the bar. We also sets standards for
Dargeling (West Bangal), Lahoul & legal education and grants recognition to
Universities whose degree in law will
serve as qualification for enrolment as 4. With reference to the Delimitation
an advocate. Commission, consider the following
statements:
In addition, we perform certain
representative functions by protecting 1. The orders of the Delimitation
the rights, privileges and interests of Commission cannot be challenged in
advocates and through the creation of a Court of Law
funds for providing financial assistance 2. When the orders of the Delimitation
to organise welfare schemes for them. Commission are laid before the Lok
Sabha or State Legislative Assembly,
http://www.thehindu.com/todays- they cannot effect any modifications
paper/bci-vows-serious- in the orders
action/article8254924.ece Which of the statements given above
is/are correct?
3. The act or process of fixing limits or
boundaries of territorial constituencies a) 1 Only
in a country or a province is known as b) 2 Only
delimitation. This job is carried out in c) Both 1 and 2
India by the d) Neither 1 nor 2

a) Territorial Commission Solution: c.


b) Boundary Commission
c) Committee on Delimitation in the CS(P) 2012, Mrunal
Rajya Sabha
d) None of the above The Delimitation Commission in India is
a high power body whose orders have
Solution: b. the force of law and cannot be called in
question before any court of law. These
Delimitation literally means the act or orders come into force on a date to be
process of fixing limits or boundaries of specified by President of India. The
territorial constituencies in a country or copies of its orders are laid before the
a province having a legislative body. House of People and the State legislative
The job of delimitation is assigned to a assembly concerned but no
high power body. Such a body is known modifications are permissible therein by
as Delimitation Commission or a them.
Boundary Commission.
http://pib.nic.in/newsite/PrintRelease.as
http://pib.nic.in/newsite/PrintRelease.as px?relid=136484
px?relid=136484
5. It is said that in the absence of long-
pending structural reforms, public sector
banks (PSBs) have more to contend with
than mounting Non-Performing Assets
(NPAs). Which of the following may be
these concerns?

1. The rapid emergence and adoption of


Fintech
2. Setting up of new as well as
differentiated banks
3. Staff shortage, specifically the top
and middle-rung managers, in the
near future
Select the correct answer using the code
given below:

a) 1 and 2 Only
b) 2 and 3 Only
c) 1, 2 and 3
d) 1 and 3 Only

Solution: c.

All are valid concerns for PSBs in India.

http://indianexpress.com/article/opinion
/editorials/rbi-psb-not-just-bad-debts/
1. Ibadat Khana from the Mughal Empire 1. He believed that an individual
may refer to harmed others as well as his own
religion when that individual
a) the mosque for the use of Royal honoured his religion and
Family condemned others
b) Akbars private prayer chamber 2. He established the Mauryan Empire
c) the hall in which Akbar held in the third century BCE
discussions with scholars of various Select the correct answer using the code
religions given below:
d) the room in which the nobles
belonging to different religions gathered a) 1 only
to discuss religious affairs b) 2 only
c) Both 1 and 2
Solution: c. d) Neither 1 nor 2

CS(P) 2014 Solution: b.

NCERT class7 History, Chapter4, Page Statement 2 is incorrect. The Mauryan


54: While Akbar was at Fatehpur Sikri Empire was established by
during the 1570s he started discussions Chandragupta Maurya in the 4th
on religion with the ulama, Brahmanas, Century BCE. His successor was
Jesuit priests who were Roman Bindusara who was succeeded by
Catholics, and Zoroastrians. These Ashoka.
discussions took place in the Ibadat
Khana. Emperor Ashokas Girnar Rock Edict
near Junagarh in Gujarat around the year
http://pib.nic.in/newsite/PrintRelease.as 260 BC:
px?relid=136579
The King honours all religions and
2. Akbars Ibadat Khana is located in sects. His Sacred Majesty does not value
gifts and honours as he values the
a) Agra growth of the essential elements of all
b) Fatehpur Sikri religious sects. But the root of it is
c) Delhi restraint of speech, that is, there should
d) Lahore not be honour only of ones own religion
and condemnation of other religions. On
Solution: b. the other hand, other religions should be
honoured too. By doing this, one helps
Read explanation above. his own religion to grow and benefits the
religion of others also. By doing
http://pib.nic.in/newsite/PrintRelease.as otherwise, one harms his own religion
px?relid=136579 and injures the other religions too. For
whoever honours only his own religion
3. With reference to Emperor Ashoka, and condemns other religions injures
which of the following statements is/are more gravely his own religion. Hence
incorrect? concord alone is commendable and all
should listen, and be willing to listen, to to eliminate any remaining threat in a
the beliefs professed by others. volatile area
c) A show of the might of a government
http://pib.nic.in/newsite/PrintRelease.as with the help of its armed forces a
px?relid=136579 concept that was introduced by the
British
4. During the second half of the nineteenth d) None of the above
century, modern industry began to take
root in India but its progress remained Solution: a.
very slow. In this context, which of the
following statements is/are incorrect? What is a flag march?

1. Initially, this development was It is an ancient concept of the Kings who


confined to the setting up of jute and used their armies to suppress difficult
cotton textile mills law and order situations in their
2. In the early years of the modern kingdoms. King Ashoka and rulers in
industry in India, the cotton textile Mughal and medieval periods in India
mills were mainly dominated by are said to have used their armies to quell
Indians while the jute mills were civil up-risings in their territories. This
dominated by the foreigners worldwide concept had been frequently
Select the correct answer using the code used by the British, first against their
given below: own people during the Tudor period in
the 15th century, and then more freely,
a) 1 Only in their colonies to suppress revolts
b 2 Only organised by natives against the British
c) Both 1 and 2 rule. During the struggle for
d) Neither 1 nor 2 independence in India, these marches
were often conducted by the British
Solution: d. Indian army.

Both statements are correct. The Army conducted a flag march in


tense areas of the State and used
Page 7 Indian Economic Development helicopters to reach parts of Rohtak
Class XI NCERT district.

http://pib.nic.in/newsite/PrintRelease.as http://www.thehindu.com/todays-
px?relid=136566 paper/blockades-force-army-to-airdrop-
troops/article8262745.ece
5. A flag march is
6. Consider the following statements:
a) an ancient concept of the Kings who
used their armies to suppress difficult Assertion (A): Many victims of extreme
law and order situations in their hypothermia peel off most or all of their
kingdoms clothing
b) another term for combing
operations used by armed forces today
Reason (R): Extreme hypothermia 7. Which of the following statements about
causes sudden, uncontrolled and the Bacillus CalmetteGurin (BCG)
intermittent spikes in body temperature vaccine is/are incorrect?

Select the correct answer: 1. It is a vaccine primarily used against


tuberculosis (TB), and is also used as
a) A and R both are true, and R is the an additive in a number of cancer
correct explanation for A vaccines
b) A and R both are true, and R is the 2. The BCG vaccine, in use at present,
NOT the correct explanation for A can only protect against severe forms
c) A is incorrect, R is correct of the disease in children but cannot
d) A is correct, R is incorrect prevent pulmonary TB in all age
groups, including children
Solution: d. Select the correct answer using the code
given below:
The term paradoxical undressing
describes the behavior among many a) 1 Only
victims of extreme hypothermia of b) 2 Only
peeling off most or all of their clothing, c) Both 1 and 2
increasing heat loss. d) Neither 1 nor 2

To shut down the loss of heat from the Solution: d.


extremities, the body induces
vasoconstriction, the reflexive Both statements are correct.
contraction of blood vessels. Over time,
however, the muscles necessary for BCG vaccine
inducing vasoconstriction become
exhausted and fail, causing warm blood All eyes are on a new BCG-based TB
to rush from the core to the extremities. vaccine, VPM1002, which has shown
This results in a kind of hot flash that promise in animal and small-scale
makes victims of severe hypothermia human trials. Besides better efficacy,
who are already confused and this recombinant BCG vaccine has been
disoriented feel as though theyre found to be superior safety-wise. Unlike
burning up, so they remove their clothes, the currently used vaccine that causes
researchers have concluded. BCG-related disease in HIV-positive
babies (as they have reduced immunity),
When the rescue teams found him, he the recombinant vaccine is expected to
was wearing nothing but his thermals. be safe in this population.
He was found in a sitting position, says
Lt. Gen. S.D. Duhan, Director and http://www.thehindu.com/opinion/op-
Commandant of the Army hospital. ed/new-bcgbased-tb-vaccine-revamped-
safer-and-with-greater-
http://www.thehindu.com/opinion/op- punch/article8262023.ece
ed/siachen-avalanche-fortyeight-hours-
to-martyrdom/article8262020.ece 8. Which of the following is NOT one of
the recommendations of the Fourteenth
Finance Commission?
http://www.thehindu.com/business/Eco
a) Distribution of the net proceeds of nomy/experts-want-body-to-oversee-
taxes of the Union for vertical as well as budget/article8262005.ece
horizontal devolution
b) Constitution of an Independent 9. With reference to the FRBM Act, which
Fiscal Council that objectively evaluates of the following statements is/are
budget announcements and forecasts incorrect?
c) In understanding the States needs, it
has ignored the Plan and non-Plan 1. The thirteenth finance commission
distinctions had suggested setting up of a body
d) Distribution of grants to States for which would undertake an ex ante
local bodies using 1971 population data analysis of the financial implications
of budget proposals
Solution: d. 2. At present, the Comptroller Auditor
General periodically monitors the
FFC has recommended distribution of compliance by the Government with
grants to States for local bodies using the provisions of the Act (ex-post
2011 population data with weight of review)
90% and area with weight of 10%. Select the correct answer using the code
given below:
Any Finance Commission is required to
recommend the distribution of the net a) 1 Only
proceeds of taxes of the Union between b) 2 Only
the Union and the States (commonly c) Both 1 and 2
referred to as vertical devolution); and d) Neither 1 nor 2
the allocation between the States of the
respective shares of such proceeds Solution: d.
(commonly known as horizontal
devolution). In recommending Both statements are correct. The
horizontal distribution, the FFC has used Thirteenth FC too had recommended a
broad parameters of population (1971) body similar to what the FFC has
and changes of population since, income recommended an Independent fiscal
distance, forest cover and area. council.

Members of the XIV Finance Source (No need to read this, it is the
Commission have questioned the entire 13th FC report, source is just
governments failure to act on its being provided here): Recommendation
recommendation to constitute an #32, Summary of Recommendations
Independent Fiscal Council that
objectively evaluates budget In 2012, the FRBM Act was amended to
announcements and forecasts, stressing allow the CAG the periodically monitor
that such an institution was critical to the compliance with the provisions of
improve the governments credibility on the Act. But the FFC found this
fiscal management. inadequate this was an ex-post review;
what is needed is an ex-ante review.
Salient Recommendations
http://www.thehindu.com/business/Eco
nomy/experts-want-body-to-oversee-
budget/article8262005.ece

http://swarajyamag.com/economy/stron
g-frbm-act-is-the-time-right-to-loosen-
the-chastity-belt

10.What is a Bad Bank, often mentioned


in the news?

a) An Asset Reconstruction Company


b) A bank which has more than 5
percent of its loans categorized as Non-
Performing Assets
c) Another name for Public Sector
Banks in India
d) A Non-Banking Financial Company
which has more than 5 percent of its
loans categorized as Non-Performing
Assets

Solution: a.

.Even as mounting bad loans have


put Indian banks in a pincer-like
situation, the idea of a government-
backed bad bank has kicked off quite a
debate. Bad bank concept allows a
government-supported entity to buy bad
loans from stressed banks at a fair price
(discount). Such an entity will then be
responsible for recovering the debt.

http://www.thehindu.com/business/Indu
stry/banks-require-holistic-
cure/article8262010.ece
1. Which of the following is a provision c) The ratio of persons unemployed to
made by Section 124A of the Indian those employed
Penal Code? d) Both a and b

a) Whoever speaks against the Solution: a.


government has committed the offence
of sedition Unemployment rate is defined as the
b) Whoever means to excite or attempts number of persons unemployed as a
to excite disaffection (among others, by proportion of the labour force (persons
words spoken or written) towards the who are either working or seeking or
nation has committed the offence of available for work), not the total
sedition population or the working age
c) Whoever promotes enmity between population.
different groups on grounds of religion
shall be punished with imprisonment, or http://www.thehindu.com/data/unemplo
with fine, or with both yment-down-in-urban-centres-but-
d) None of the above persists-in-rural-areas-says-
survey/article8264731.ece
Solution: d.
3. Consider the following statements with
Section 124A of the IPC says, says: regard to the findings of the recently
released National Sample Survey
Whoever by words either spoken or Offices employment and
written or by signs or by visible unemployment survey:
representation or otherwise brings or
attempts to bring into hatred or 1. Self-employment is the major source
contempt, the government established of income for almost half the
by law; or households, across all religious
Whoever by the above means excites or groups, in rural areas
attempts to excite disaffection towards 2. In urban areas, the proportion of
the government established by law (and households deriving major income
NOT towards the nation), has committed from regular wage or salary earnings
the offence of sedition. is the highest
General Question based on the ongoing Which of the statements above is/are
debates/discussions on Section 124A of correct?
IPC.
a) 1 Only
2. Unemployment Rate is defined as b) 2 Only
c) Both 1 and 2
a) The number of persons unemployed d) Neither 1 nor 2
as a proportion of the labour force
(persons who are either working or Ans: c.
seeking or available for work)
b) The number of persons unemployed http://www.thehindu.com/data/unemplo
as a proportion of the working age yment-down-in-urban-centres-but-
population persists-in-rural-areas-says-
survey/article8264731.ece
persists-in-rural-areas-says-
4. Consider the following statements: survey/article8264731.ece

Assertion (A): Christians in India have 5. Urur Olcott Kuppam Vizha is


the highest rate of unemployment in
both rural and urban areas a) A music festival which showcases
Keralas tribal folk music
Reason (R): Unemployment level in b) An art festival associated with the
India is highest among those people who fisher folk of a village in Tamil Nadu
are richer and more educated c) A dance festival associated with the
Araiyar community of Muttukuru in
Select the correct answer: Andhra Pradesh
d) A celebration to mark the beginning
a) A and R both are true, and R is the of the fishing season in the coastal city
correct explanation for A of Udupi in Karnataka
b) A and R both are true, and R is the
NOT the correct explanation for A Solution: b.
c) A is incorrect, R is correct
d) A is correct, R is incorrect http://www.thehindu.com/opinion/op-
ed/chennai-art-festival-from-sabhas-to-
Ans: a. fishing-villages/article8264553.ece

Christians have the highest rate of 6. The Ministry of Environment and


unemployment in both rural (4.5 per Forests has extended the general
cent) and urban (5.9 per cent) areas in approval for creation of critical public
2011-12. The rate in urban areas for utility infrastructure by government
Christians stood at 8.6 per cent in 2004- departments, in the Left Wing
05 while the rural rate stays constant. Extremism (LWE) affected areas, up to
2018. Under which act has this general
Speaking to The Hindu, Alakh approval been accorded?
Sharma, Director of Institute of Human
Development, said: Unemployment a) The Forest Rights Act 2006
level in India is highest among those b) Forest (Conservation) Act, 1980
people who are richer and more c) Wildlife Protection Act, 1972
educated. The reason is that poor people d) The Environment Protection Act,
cant afford to stay unemployed, and 1986
hence, opt for any kind of work,
irrespective of the nature of the job. The Solution: b.
better off have the capacity to be
unemployed as they look for the right Realising the importance of
job. Christians are the most educated developmental activities in the Left
group, hence unemployment rate is Wing Extremism (LWE) affected areas,
higher among them. Data from the the general approval as contained in the
report supports the claim Ministry of Environment and Forest
letter, dated 13th May, 2011, has been
http://www.thehindu.com/data/unemplo extended up to December 31, 2018, as
yment-down-in-urban-centres-but- per the extant terms and conditions.
In an effort to expedite the creation of
public utility infrastructure in 60 Left
Wing Extremism (LWE) affected
districts identified by the Planning
Commission for implementation of the
Integrated Action Plan (IAP), the
Ministry of Environment & Forests, vide
letter dated May 13, 2011, had accorded
general approval under section 2 of
Forest (Conservation) Act, 1980, for
creation of critical public utility
infrastructure by government
departments, not involving more than
5.00 hectares of forest land. This
provision was further extended to a total
of 117 LWE affected districts. The
approval was valid upto December 31,
2015.

http://pib.nic.in/newsite/PrintRelease.as
px?relid=136612
1. The Banaras Hindu University (BHU) Solution: d.
recently held its centenary year
celebrations. In this context, which of All statements are correct.
the following statements is/are
incorrect? CERN is the worlds largest body of
experts in nuclear and particle physics,
1. Gandhijis first major public where scientists and engineers are
appearance in India was at the probing the fundamental structure of
opening of the BHU universe by using the most sophisticated
2. BHUs inauguration marked the scientific instruments and advanced
opening of a nationalist university, systems. CERN was established in 1954
sustained by Indian money and and is based in Geneva. Significantly,
Indian initiative CERN is the birthplace of www
Select the correct answer using the code (World Wide Web) which is regularly
given below: used to browse the internet. Similarly,
the touch screen widely used in
a) 1 Only smartphones was also first developed by
b) 2 Only CERN.
c) Both 1 and 2
d) Neither 1 nor 2 http://pib.nic.in/newsite/PrintRelease.as
px?relid=136644
Solution: d.
3. An application to become an Associate
Page 347, 348 Themes in Indian Member (AM) of CERN has been
History Part III Class XII NCERT submitted by the Department of Atomic
Energy, Government of India. In this
http://pib.nic.in/newsite/PrintRelease.as context, which of the following
px?relid=136637 statements is/are correct?

2. Which of the following statements about 1. Pakistan is already an AM of CERN


CERN is/are correct? 2. AMs dont have to contribute to the
CERN budget
1. It stands for European Organisation 3. AMs have reduced voting rights in
for Nuclear Research comparison to Members of CERN
2. It is the birthplace of www the Select the correct answer using the code
World Wide Web given below:
3. The touch screen widely used in
smartphones was also first developed a) 1 Only
by it b) 1 and 3 Only
Select the correct answer using the code c) 1 and 2 Only
given below: d) 1, 2 and 3

a) 2 and 3 Only Solution: a.


b) 1 and 3 Only
c) 1 and 2 Only 1.2 Associate Membership
d) 1, 2 and 3
This status can either take the form of The National Election Commission of
regular Associate Membership (of the Republic of Korea (NEC) proposed
indefinite duration) or of Associate its establishment.
Membership as the pre-stage to
Membership, normally for a period of Association of World Election Bodies
between two and five years. Associate was established in 2013 and has 106
Members pay a reduced contribution to members from 102 countries. The
the CERN budget and enjoy benefits members comprise national Election
which are reduced accordingly. They are Management Bodies and some regional
represented at the CERN Council, associations and international electoral
except at Closed Sessions, and do not organisations. A-WEBs vision is to
have voting rights. CERN currently has foster efficiency and effectiveness in
one Associate Member in the pre-stage conducting free, fair, transparent and
to Membership, Serbia and two participative elections worldwide. Its
Associate Members, Pakistan and activities are guided by its mission to
Turkey. identify latest trends, challenges and
developments in democratic electoral
The major advantage of the associate management and electoral processes and
membership of CERN is that this will to facilitate appropriate exchange of
entitle India to become a part of huge experience and expertise among
scientific and technological endeavour. members with the objective of
This will also offer an opportunity to strengthening electoral democracy
Indian industry to bid for CERN worldwide.
contracts for industrial collaboration in
advanced technologies. http://pib.nic.in/newsite/PrintRelease.as
px?relid=136639
http://pib.nic.in/newsite/PrintRelease.as
px?relid=136644 5. Which of the following statements is/are
incorrect?
4. Which of the following statements about
the Association of World Election 1. The division of India into a number
Bodies (A-WEB) is NOT correct? of seismic zones is termed Seismic
Microzonation
a) It provides training for election 2. The impact of an earthquake does not
officials necessarily decrease as one moves
b) The Election Commission of India away from its epicenter
spearheaded its establishment Select the correct answer using the code
c) It organizes various election given below:
observations
d) Its members comprise some regional a) 1 Only
associations and international electoral b) 2 Only
organisations c) Both 1 and 2
d) Neither 1 nor 2
Solution: b.
Solution: a.
Earlier, a seismic zoning map for entire c) Both 1 and 2
India was prepared and published by the d) Neither 1 nor 2
Bureau of Indian Standards (BIS),
classifying the entire country into 4 Solution: a.
major groups Zone-V (High intensity) to
Zone-II (Low intensity). This showed As of February 23rd 2016, Britain is still
the macro seismic zones. a member of the EU.

It was seen that due to an earthquake in That Britain is in a minority of


1985 there was severe damage in countries that neither share the euro nor
Mexico city, though the earthquake participate in the Schengen border-free
source was located about 350 km away zone does not diminish its weight and
from the city. It was realized that putting importance in the larger EU
a large region into a single seismic framework.
hazard zone is not a good idea. This and
several other examples led to a new http://www.thehindu.com/opinion/edito
concept, called Microzonation. Seismic rial/referendum-gamble-for-
microzonation is a process of identifying britain/article8268210.ece
such related geological, seismological,
hydrological and geotechnical site 7. Consider the following statements:
characteristics in a specific region.
These would help design of safe a. River Bhadra originates from here
structures to reduce loss of human life. b. It consists of montane grasslands as
well as tropical rainforests
According to Wikipedia, In most c. This is a part of a UNESCO World
general terms, seismic microzonation is Heritage Site
the process of estimating the response of The above statements refer to
soil layers under earthquake excitations
and thus the variation of earthquake a) Great Himalayan National Park
characteristics on the ground surface Conservation Area
b) Kudremukh National Park
http://pib.nic.in/newsite/PrintRelease.as c) Kaziranga National Park
px?relid=136632 d) Manas Wildlife Sanctuary

6. Consider the following statements about Solution: b.


Britain and its relationship with the
European Union (EU): http://www.thehindu.com/opinion/colu
mns/disappearing-forests-of-
1. Britain is a member of the EU kudremukh/article8268142.ece
2. Britain does not share the Euro but is
a part of the Schengen border-free
zone
Which of the statements above is/are
correct?

a) 1 Only
b) 2 Only
1. An exhibition on the Coin and hills etc. and some were geometrical
Currency of India was recently symbols.
organised by the Department of
Expenditure, Ministry of Finance. https://www.rbi.org.in/currency/museu
Consider the following statements m/c-ancient.html
regarding coinage in India:
2. Under what circumstances is the Army
1. The earliest of documented coinage requisitioned by the civilian
is deemed to start with coins issued administration?
around the 3rd century BCE
2. The earliest coins were mostly made a) Maintenance of law and order
of silver and were punch-marked b) Maintenance of essential services
3. Coins are considered to have been c) Any type of assistance which may be
issued initially by merchant guilds needed by the civil authorities
and later by states d) All of the above
Which of the statements above is/are
correct? Solution: d.

a) 1 and 3 Only Regulations permit civil authorities to


b) 2 Only requisition the Army for (1)
c) 1 and 2 Only maintenance of law and order, (2)
d) 2 and 3 Only maintenance of essential services, (3)
assistance during natural calamities such
Solution: d. as earthquakes, (4) any other type of
assistance which may be needed by the
This Exhibition depicts how money has civil authorities.
changed in appearance and substance
and evolved from barter to coins to the http://indianexpress.com/article/explain
present day bank notes and e-money. ed/jat-agitation-army-in-aid-of-civil-
authority-when-how-do-troops-come-
The first documented coinage is in/
deemed to start with Punch Marked
coins issued between the 7th-6th century 3. The International Roma Conference and
BC and 1st century AD. These coins are Cultural Festival was recently took place
called punch-marked coins because of in Delhi. In this context, which of the
their manufacturing technique. Mostly following statements about Roma is
made of silver, these bear symbols, each correct?
of which was punched on the coin with
a separate punch. a) Roma in Greek refers to a tribal
people who are known to have migrated
Issued initially by merchant Guilds and to different parts of Europe, Asia and
later by States, the coins represented a Africa
trade currency belonging to a period of b) It means fusion food and the
intensive trade activity and urban conference celebrates the diverse foods
development. The motifs found on these of our world
coins were mostly drawn from nature
like the sun, various animal motifs, trees,
c) They are a travelling people who live http://www.livemint.com/Opinion/brbD
mostly in Europe and America, and 6Tw1akpGS3um0t3CaM/India-is-an-
whose origins lie in India outlier-in-its-tax-policy.html
d) From the Greek Roma was derived
the English word aroma; the festival is 5. Which of the following statements
a celebration of different brews in coffee associated with Aerosols is/are correct?
of the world
1. Aerosol Optical Depth is a measure
Solution: c. of the extinction of the solar beam by
dust and haze
The International Roma Conference and 2. Aerosols can either cool the surface,
Cultural Festival was organised recently or warm it
by International Council for Cultural Select the correct answer using the code
Relations, the cultural wing of MEA, given below:
and Antar Rashtriya Sahayog Parishad-
Bharat. a) 1 Only
b) 2 Only
http://indianexpress.com/article/explain c) Both 1 and 2
ed/meet-the-roma-2000-years-ago-the- d) Neither 1 nor 2
first-indians-to-go-to-europe/
Solution: c.
4. In economics, the Laffer curve is
Aerosol optical depth or optical
a) a representation of the relationship thickness is a measure of the extinction
between rates of taxation and the of the solar beam by dust and haze. In
resulting levels of government revenue other words, particles in the atmosphere
b) indicates that there is a level of (dust, smoke, pollution) can block
government spending that maximises sunlight by absorbing or by scattering
economic growth - the optimal level of light. AOD tells us how much direct
government spending accordingly is 15 sunlight is prevented from reaching the
25% of GDP ground by these aerosol particles. It is a
c) It organizes various election dimensionless number that is related to
observations the amount of aerosol in the vertical
c) is a graphical representation of the column of atmosphere over the
distribution of income, viz. representing observation location.
inequality of wealth distribution
A value of 0.01 corresponds to an
Solution: a. extremely clean atmosphere, and a value
of 0.4 would correspond to a very hazy
We also know that there is a maximum condition.
feasible tax-to-GDP ratio, which
corresponds to the peak of the celebrated Tiny solid and liquid particles suspended
Laffer curve, but this is only an upper in the atmosphere are called aerosols.
bound on the governments taxing Windblown dust, sea salts, volcanic ash,
capacity, again not a statement of what smoke from wildfires, and pollution
is the optimum. from factories are all examples of
aerosols. Depending upon their size,
type, and location, aerosols can either 7. Consider the following statements:
cool the surface, or warm it. They can
help clouds to form, or they can inhibit 1. Fiscal metrics in India are weak when
cloud formation. And if inhaled, some compared to other countries with
aerosols can be harmful to peoples similar sovereign credit ratings
health. 2. India, compared to its peers, relies
lower on foreign currency debt one
http://www.thehindu.com/opinion/edito reason being its public debt-to-GDP
rial/air-pollution-in-india-clean-air- ratio is lower than similarly rated
agenda-for-the- countries
cities/article8272573.ece Which of the statements above is/are
correct?
6. Which of the following has/have been
accorded Geographical Indication a) 1 Only
status? b) 2 Only
c) Both 1 and 2
1. Basmati d) Neither 1 nor 2
2. Nizamabad Black Pottery
3. Meghalayas Memong Narang Solution: a.
Select the correct answer using the code
given below: The rating agency did highlight one
silver lining for India compared to its
a) 1 and 3 Only peers lower reliance on foreign
b) 2 and 3 Only currency debt, even though its public
c) 1 and 2 Only debt to GDP ratio is higher than
d) 1, 2 and 3 similarly rated countries like Indonesia,
Philippines, Romania and Turkey.
Solution: d.
http://www.thehindu.com/business/Eco
Each of the above-mentioned nomy/fiscal-metrics-remain-weak-
goods/produce has been granted GI moodys/article8272615.ece
status post-April 2015.

Click on registered GIs link in the list


on the left hand side on this page.

http://indianexpress.com/article/explain
ed/whats-the-row-robbing-basmati-of-
its-aroma/
1. Certain medicines will soon carry a red
line on their packaging, differentiating Merchant Discount Rate: The rate
them from other drugs. What is this charged to a merchant by a bank for
coloured line supposed to convey? providing debit and credit card services.
The rate is determined based on factors
a) The drug is on the National List of such as volume, average ticket price,
Essential Medicines risk and industry. The merchant must set
b) That the drug must be used carefully, up this service with a bank, and agree to
and to discourage unnecessary the rate prior to accepting debit and
prescription and over-the-counter sale credit cards as payment.
c) The drug is a generic drug
d) That the drug is manufactured by http://pib.nic.in/newsite/PrintRelease.as
Public-Private Partnership Enterprises px?relid=136755

Solution: b. 3. Associated with Indian History, the


Battle of Aberdeen refers to
http://pib.nic.in/newsite/PrintRelease.as
px?relid=136657 a) Vanquish of the Dutch from
Travancore by the British in the 18th
century
b) A battle fought by the Andamanese
http://timesofindia.indiatimes.com/india aborigines and British
/Govt-draws-thin-red-line-to-curb- c) The last battle fought by Indian
antibiotics- soldiers in World War II
misuse/articleshow/51114973.cms d) One of the many engagements in the
wars fought among the Portuguese,
2. Consider the following statements: French and the British on Indian lands

1. The penetration and success of Solution: b.


modern card/digital payment
products and services is concentrated The Andaman & Nicobar
to a large extent in the tier-l and tier- Administration has apprised that a
ll locations of the country Memorial has been constructed at the
2. Merchant Discount Rate is the rate entrance of the Water Sports Complex at
charged to a customer by the Port Blair in the memory of those Great
merchant on account of bank-charges Andamanese, who had laid down their
s/he bears for providing debit and lives in the famous Battle of Aberdeen
credit card services fought on 17th May 1859 against
Which of the statements above is/are Britishers and a solemn function is
correct? organized every year on this day.

a) 1 Only http://pib.nic.in/newsite/PrintRelease.as
b) 2 Only px?relid=136699
c) Both 1 and 2
d) Neither 1 nor 2

Solution: a.
4. Agitations for Cooch Behar to be http://indianexpress.com/article/explain
designated as a C category state have ed/simply-put-whats-behind-the-
been ongoing. What does this C protests-rail-roko-in-north-bengal/
category state mean?

a) An entirely separate state


b) A centrally administered
division/state
c) Any state with its own Constitution
and Flag
d) None of the above

Solution: b.

There is a demand that the Greater


Cooch Behar area be recognised either
as a separate C category state or a
Union Territory. [These Part-C states
were centrally administered Page 5.2
Indian Polity 4th Edition by M.
Laxmikanth]

The demand for a separate Greater


Cooch Behar, comprising Cooch Behar,
Darjeeling, Jalpaiguri, North and South
Dinajpur, and the undivided Goalpara
district of Assam, is based on the claim
that the original merger agreement
between the Maharaja of Cooch Behar
and the Government of India was
ignored during Cooch Behars inclusion
into India in 1949.

Cooch Behar was a princely state until it


became a part of India on August 20,
1949, and the GCPA argues that its
inclusion in West Bengal violated the
terms of that agreement. The GCPA
maintains that the original agreement
between the king of Cooch Behar and
the Indian government promised Cooch
Behar C category statehood, but it was
merged with West Bengal an A
category state.
1. Consider the following statements:
2. Before the Socio-Economic Caste
1. Advertising is a source of Non-fare Census (SECC) 2011, the only effort at
box revenue for the Indian Railways collecting data on different castes and
(IR) their socio-economic circumstances was
2. A lower figure of Operating Ratio is undertaken by the
indicative of better financial health of
a system such as the IR a) Census 2001
Which of the statements above is/are b) Census 1931
correct? c) SECC 1981
d) Census 1971
a) 1 Only
b) 2 Only Solution: b.
c) Both 1 and 2
d) Neither 1 nor 2 Since the 1931 Census, the only effort
at collecting data on different castes and
Solution: c. their socio-economic circumstances was
undertaken by the Socio-Economic
Railway Minister Suresh Prabhus Caste Census (SECC), 2011. The
projection of savings of Rs.8,720 crore National Commission for Backward
compared with budget estimates for the Classes claimed, in a report dated
current fiscal year reflects a finance February 2015, that these data are
professionals approach in adopting neither available nor usable for the
austerity measures to contain costs. purpose of establishing the economic
Building on those gains, the budget has condition of various castes.
projected that notwithstanding the
substantial jump in salary and pension http://www.thehindu.com/opinion/op-
costs consequent upon the ed/caste-reservation-quantifying-the-
implementation of the Seventh Pay caste-quotes/article8281303.ece
Commissions recommendations, the
impact would be minimised to an 11.6 3. The Office of the Controller General of
per cent increase in working expenses Patents, Designs & Trade Marks comes
next year. This will lead to a two under
percentage point rise in the operating
ratio. a) Ministry of Commerce and Industry
b) Ministry of Law and Justice
http://indianexpress.com/article/explain c) Ministry of Science and Technology
ed/railway-budget-non-tariff-revenue- d) Ministry of Corporate Affairs
or-non-fare-box-revenue-suresh-
prabhu/ Solution: a.

The Office of the Controller General of


Patents, Designs & Trade Marks comes
http://www.thehindu.com/opinion/edito under the Department of Industrial
rial/railway-budget-2016-marshalling- Policy and Promotion, Ministry of
resources-to-stay-on- Commerce and Industry.
track/article8281274.ece
http://www.thehindu.com/opinion/edito backward castes, resulting in frequent
rial/can-computer-programmes-be- discordance between these lists.
granted-patents/article8281275.ece
http://www.thehindu.com/opinion/op-
4. Consider the following statements: ed/caste-reservation-quantifying-the-
caste-quotes/article8281303.ece
1. In India, software in itself is not
patentable
2. Patent thicket is a term used by the
developed nations to denote what
they perceive to be rigid patent
regimes such as Indias
Which of the statements above is/are
correct?

a) 1 Only
b) 2 Only
c) Both 1 and 2
d) Neither 1 nor 2

Solution: a.

In the U.S., a more open policy has led


to a flood of patents, and consequently
the negative connotation that the term
patent thicket carries now.

http://www.thehindu.com/opinion/edito
rial/can-computer-programmes-be-
granted-patents/article8281275.ece

5. The Mandal Commission was the


Second Backward Class Commission.
The First Backward Class Commission
was the

a) Kaka Kalelkar Commission


b) Morarji Desai Commission
c) Shastri Commission
d) Jagjivan Ram Commission

Ans: a.

Since the First Backward Classes


Commission headed by Kaka Kalelkar
submitted its report in 1955, several
attempts have been made to identify
1. What, according to the latest Economic government workers if the seventh pay
Survey, are the most critical short term commission is implemented and return
challenges confronting the Indian of normal monsoon.
economy?
http://pib.nic.in/newsite/PrintRelease.as
1. Poor monsoons px?relid=136850
2. Impaired financial position of the
Public Sector Banks 2. For addressing the most critical short
3. Impaired financial position of some term challenges confronting the Indian
corporate houses economy, the latest Economic Survey
4. Impact on inflation due to recommends adopting the 4R method.
implementation of 7th Pay Which of these Rs as mentioned
Commission recommendation below, are correctly defined?
5. The turmoil in global economy
Select the correct answer using the code 1. Recognition: That foreign demand is
given below: weak and hence activate domestic
sources of demand to prevent the
a) 2,3 and 4 Only growth momentum from weakening
b) 1, 2, 3 and 4 Only 2. Recapitalisation: Capital position of
c) 2 and 3 only banks must be safeguarded via
d) 1, 4 and 5 only infusions of equity
3. Resolution: Of constraints to ease of
Solution: c. doing business so as to instill
confidence in investors and
The Survey cautions that one of the subsequently attract investment
most critical short term challenges 4. Reform: The contribution of the
confronting the Indian economy is the agriculture sector to the GDP must be
twin balance sheet problem the enhanced by ushering in structural
impaired financial positions of the reforms
Public Sector Banks (PSBs) and some Select the correct answer using the code
corporate houses. The twin balance given below:
sheet challenge is the major impediment
to private investment and a full-fledged a) 1, 3 and 4 Only
economic recovery. b) 2 Only
c) 1 and 2 Only
The Survey enumerates three down d) 2, 3 and 4 Only
side risks (and not challenges) turmoil
in global economy could worsen the Solution: b.
outlook of exports, contrary to
expectations oil prices rise would Comprehensively resolving the twin
increase the drag from consumption and balance sheet challenge would require 4
the most serious risk is combination of Rs: Recognition, Recapitalization,
the above two factors. Resolution, and Reform.

On the domestic side, two factors can Banks must value their assets as far as
boost consumption, increased spending possible close to true value (recognition)
from higher wages and allowances of as the RBI has been emphasizing;
Once they do so, their capital position 3 percent of world exports, capture
must be safeguarded via infusions of nearly 15% of world market share.
equity (re-capitalisation)as the banks
have been demanding; 4. Consider the following statements:
The underlying stressed assets in the
corporate sector must be sold or 1. As an investment proposition, the
rehabilitated (resolution) as the Rational Investor Ratings Index
government has been desiring; and shows that India compares
Future incentives for the Private Sector favourably with peer countries
and corporates must be set-right 2. In India, for the last year or so, even
(reform) to avoid a repetition of the as real growth has been accelerating,
problem, as everyone has been nominal growth has been falling
clamouring. Which of the statements above is/are
http://pib.nic.in/newsite/PrintRelease.as correct?
px?relid=136850
a) 1 Only
3. Consider the following statements: b) 2 Only
c) Both 1 and 2
1. In the last two years Indian services d) Neither 1 nor 2
exports have been more affected than
Indian manufacturing exports Solution: c.
2. Currently, Indias services exports
constitutes about 3 percent of world The Survey highlights that India stands
exports out internationally as an investment
Which of the statements above is/are proposition and the Rational Investor
incorrect? Ratings Index (RIRI) shows that India
compares favourably with peer countries
a) 1 Only in the BBB investment grade and almost
b) 2 Only matches the performance of A-grade
c) Both 1 and 2 countries.
d) Neither 1 nor 2
Graphs on page 11 of Volume 1,
Solution: d. Economic Survey 2015-16:

On the external outlook, the Survey From around Q4 of Fiscal Year 2014-15,
says that overall exports declined by growth rate of GDP at constant prices
about 18% in the first three quarters. It has been gradually rising while the
points-out that in the last two years growth rate of GDP at current prices is
Indian services exports have been more declining overall (just one spike between
affected than Indian manufacturing Q2 and Q3 of FY2015-16).
exports and also world service exports.
Realizing Indias medium term growth The Survey points out that even as real
potential of 8-10 percent will require growth has been accelerating, nominal
rapid growth of export. To achieve growth has been falling, an unusual
trajectory similar to China, Indias trend highlighted in the Mid-Year
competitiveness will have to improve so Economic Analysis (MYEA), 2015-16.
that its services exports, currently about
undernourished people at 194.6 million
persons (FAO, State of Food Insecurity
in the World, 2015) which warrants
immediate attention.

http://pib.nic.in/newsite/PrintRelease.as
px?relid=136851
http://pib.nic.in/newsite/PrintRelease.as
6. With reference to the dairy sector in
px?relid=136850
India, which of the following statements
is/are correct?
5. Which of the following statements is/are
incorrect?
1. India ranks first in milk production,
accounting for close to 20 per cent of
1. Average protein intake per capita per
world production
day rises steadily with the Monthly
2. The per capita availability of milk in
Per Capita Expenditure in rural as
India is just below the world average
well as urban India
Select the correct answer using the code
2. India has the highest number of
given below:
undernourished people
Select the correct answer using the code
a) 1 Only
given below:
b) 2 Only
c) Both 1 and 2
a) 1 Only
d) Neither 1 nor 2
b) 2 Only
c) Both 1 and 2
Solution: a.
d) Neither 1 nor 2
India ranks first in milk production,
Solution: b.
accounting for 18.5 per cent of world
production, achieving an annual output
As per the Report of Nutritional Intake
of 146.3 million tones during 2014-15
in India, 2011-12 (NSSO, 68th round),
as compared to 137.69 million tonnes
among the bottom 5 per cent of rural
during 2013-14 recording a growth of
population ranked by Monthly Per
6.26 per cent. Whereas, the Food and
Capita Expenditure (MPCE), 57 per cent
Agriculture Organization (FAO) has
of households had calorie intake below
reported a 3.1 per cent increase in world
2160 Kcal/consumer unit/day. The
milk production from 765 million tones
average protein intake per capita per day
in 2013 to 789 million tones in 2014.
rises steadily with MPCE level in rural
India from 43 gm for the bottom 5 per
The per capita availability of milk in
cent of population ranked by MPCE to
India has increased from 176 grams per
91 gm for the top 5 per cent, and in urban
day in 1990-91 to 322 grams per day by
India from 44 gm for the bottom 5 per
2014-15. It is more than the world
cent to about 87 gm for the top 5 per
average of 294 grams per day during
cent.
2013.
Economic Survey 2015-16 states that
India has the second highest number of
http://pib.nic.in/newsite/PrintRelease.as
px?relid=136849 a) It gives buyers, with electricity load
above 1 Mega Watt (MW), the right to
7. 2014-15 witnessed the highest ever procure electricity directly from
increase in generation capacity of 26.5 electricity markets
GW compared to the average annual b) It gives buyers, with electricity load
addition of around 19 GW over the past above 5 MW, the right to procure
five years. In this backdrop, consider the electricity directly from electricity
following statements: markets
c) It gives households, which
1. High tariffs and erratic supply have domestically generate power, the option
led to a slow but steady decline of of storing the surplus to discoms which
industrial electricity purchases from can be acquired by the households in
utilities times of need, for free
2. Current power plant load factors have d) It gives households, which
reached their lowest levels in recent domestically generate power, the option
times of selling the surplus to discoms which
Which of the statements above is/are can be acquired by the households in
incorrect? times of need, at a subsidized price

a) 1 Only Solution: a.
b) 2 Only
c) Both 1 and 2 The Survey states that steps have been
d) Neither 1 nor 2 taken towards Making One India in
the power sector. The Open Access
Solution: a. (OA) policy introduced under the
Electricity Act 2003, which allows
High tariffs and erratic supply have led consumers with electricity load above 1
to a slow but steady decline in growth of MW to procure electricity directly from
industrial electricity purchases from the electricity market was the first step
utilities. So industrial electricity towards discovering a single market
purchases as a whole is not declining, price for power around the country.
but the growth in purchases is.
The power generation capacity has
The power generation capacity has increased while the financial ability of
increased while the financial ability of discoms to purchase electricity has
discoms to purchase electricity has diminished. This has resulted in current
diminished. This has resulted in current power plant load factors reaching their
power plant load factors reaching their lowest mark at around 60% as The
lowest mark at around 60%. Survey notes that the time is ripe to
allow industries with higher power
http://pib.nic.in/newsite/PrintRelease.as demands to absorb excess generation
px?relid=136848 capacity through Open Access to
energize Make in India
8. An Open Access (OA) policy was
introduced under the Electricity Act http://pib.nic.in/newsite/PrintRelease.as
2003. What is the purpose of this policy? px?relid=136848
1. The right to raise a question of privilege a) 1 Only
shall NOT be governed by which of the b) 2 Only
following conditions? c) Both 1 and 2
d) Neither 1 nor 2
1. Not more than one question shall be
raised at the same sitting Solution: c.
2. The question shall be restricted to a
specific matter of recent occurrence The Attorney General of India is entitled
Select the correct answer using the code to speak and take part in the proceedings
given below: of a house of the Parliament or any of its
Committees. The Constitution extends
a) 1 Only Parliamentary Privileges to the AGI and
b) 2 Only members of such committees as well.
c) Both 1 and 2
d) Neither 1 nor 2 Page 22.34 Indian Polity 4th Edition by
M Laxmikanth
Solution: d.
http://www.thehindu.com/news/national
The right to raise a question of privilege /congress-to-move-privilege-motion-
shall be governed by the following against-smriti-irani/article8290288.ece
conditions, namely:
3. Consider the following statements:
not more than one question shall be
raised at the same sitting; Assertion (A): Coral reefs are vulnerable
the question shall be restricted to a to ocean acidification
specific matter of recent occurrence; and
the matter requires the intervention of Reason (R): Any rise in acidity of water
the House. increases the susceptibility of coral
http://www.thehindu.com/news/national skeleton to dissolve in water
/congress-to-move-privilege-motion-
against-smriti-irani/article8290288.ece Select the correct answer:

2. With reference to Parliamentary a) A and R both are true, and R is the


Privileges, which of the following correct explanation for A
statements is/are correct? b) A and R both are true, and R is the
NOT the correct explanation for A
1. These privileges are available not c) A is incorrect, R is correct
only to members of the Parliament d) A is correct, R is incorrect
2. Parliamentary Privileges do not
extend to the President who is also an Ans: d.
integral part of the Parliament
Select the correct answer using the code Ocean acidification arises when nearly
given below: 25 per cent of carbon dioxide released
into the atmosphere and absorbed by the
oceans reacts with water to form
carbonic acid. The carbonic acid thus
produced leads to ocean acidification by
decreasing the pH of the ocean, reducing
the concentration of carbonate ion The mosquitoes thus remain unharmed
(which is essential for organisms such as by HIV. Not only this, they can not
corals and clams to build their shells and transfer the HIV into any person as they
skeletons) and a decreased aragonite have no mechanism to inject back the
mineral saturation state (a measure of HIV from their guts to human body. On
the availability of dissolved carbonate the other hand, the Zika virus is a
and calcium ions). mosquito-transmitted infection related
to dengue, yellow fever and West Nile
http://www.thehindu.com/sci- virus.
tech/landmark-experiment-improves-
coral-calcification-by- NCERT 12th Class Biology: HIV enters
7/article8289731.ece into helper T-lymphocytes (TH),
replicates and produce progeny viruses.
4. Consider the following statements: The progeny viruses released in the
blood attack other helper T-
Assertion (A): Mosquitoes are lymphocytes. This is repeated leading to
unharmed by and unable to transmit HIV a progressive decrease in the number of
virus helper T-lymphocytes in the body of the
infected person. During this period, the
Reason (R): T-cells or T-lymphocytes person suffers from bouts of fever,
are absent in a mosquitos digestive diarrhoea and weight loss. Due to
system decrease in the number of helper T
lymphocytes, the person starts suffering
Select the correct answer: from infections that could have been
otherwise overcome such as those due to
a) A and R both are true, and R is the bacteria especially Mycobacterium,
correct explanation for A viruses, fungi and even parasites like
b) A and R both are true, and R is the Toxoplasma. The patient becomes so
NOT the correct explanation for A immuno-deficient that he/she is unable
c) A is incorrect, R is correct to protect himself/herself against these
d) A is correct, R is incorrect infections.

Ans: a.

Why does a mosquito get infected by


Zika virus while it is unharmed by HIV
and other deadly viruses?

The blood sucked by a mosquito goes


straight into its digestive system and not
in its blood stream. Unlike mosquito
borne diseases, HIV is unable to
replicate within the mosquitos gut as it
has no T cells over there and therefore is
broken down very soon. HIV must have
blood as medium to further proliferate.
5. Article 130 of the Indian Constitution http://www.thehindu.com/opinion/edito
states that rial/reworking-the-supreme-courts-
role/article8292595.ece
a) The Supreme Court (SC) may, in its
discretion, grant special leave to appeal 6. Which of the following statements about
from any judgment in any cause or the Comptroller and Auditor General of
matter passed or made by any court or India is/are incorrect?
tribunal
b) The SC shall sit in Delhi or in such 1. He is appointed by the President of
other place or places, as the Chief Justice India
of India (CJI) may, with the approval of 2. He is not eligible for further office
the President, from time to time, appoint either under the Government of India
c) The CJI may at any time, with the or of any State after he has ceased to
previous consent of the president, hold his office
request a former Judge of the SC to sit Select the correct answer using the code
and act as a Judge of the SC given below:
d) An appeal shall lie to the SC from
any judgment, if the High Court certifies a) 1 Only
that the case involves a substantial b) 2 Only
question of law as the interpretation of c) Both 1 and 2
this Constitution d) Neither 1 nor 2

Solution: b. Solution: d.

The Union Law Ministry has obtained Congress spokesperson Randeep


legal opinion that a Constitution Singh Surjewala told The Hindu that
amendment to revisit the Supreme since Mr. Rai had retired as the CAG, he
Courts role would be impermissible as could not be rewarded with a new
it would change the courts character government job. Does it not violate
under the Constitution. The opinion Article 148(4) of the Constitution, Mr.
appears to disfavour a suggestion by the Surjewala asked, adding the CAG was
Law Commission in its 229th Report not eligible to take any post-retirement
(2009) that if necessary Article 130 job under the government
(The Supreme Court shall sit in Delhi
or in such other place or places, as the Chapter 45 Indian Polity 4th Edition by
Chief Justice of India may, with the M Laxmikanth
approval of the President, from time to
time, appoint) may be amended to http://www.thehindu.com/news/national
implement its suggestion that Cassation /rais-appointment-is-unconstitutional-
Benches may be set up in four regions, congress/article8293333.ece
while the Constitution Bench sits in
Delhi. Clarification regarding Question 5, Quiz
56:
Article 136 is the option a.
This is what the State of Food Insecurity
in the World, 2015 report of the Food
and Agriculture Organisation (which has
been used by the Economic Survey) has
to say about undernourishment in India
and China:

Most countries in Southern Asia have


made progress towards the international
hunger targets, even if the pace has been
too slow for them to reach either the
WFS or the MDG targets, including, for
example, Afghanistan, India, Pakistan
and Sri Lanka. As these countries
constitute a large share of the regions
population, they account for the low
overall performance India still has the
second-highest estimated number of
undernourished people in the
world. China is still home to an
estimated 134 million people facing
hunger, and the country with the highest
number of undernourished people.

So the answer for question 5 is correct


(viz. as asked in the question, statement
2 only is incorrect). The Economic
Survey (along with the corrigendum)
one can consider the data/stats/facts
given in it to be gospel truth. Do NOT go
wrong with such data/stats/facts.

You might also like